You are on page 1of 94

Lecture Notes On Calculus

>>>>>>>>>>>>>>>>>> M.A.HALIM<<<<<<<<<<<<<<<<<<<<<<

DIU
SEMESTER | SPRING-18
Chapter 01 : Function and its various types
The mathematical term function was included to this subject about 17 century to express the
dependency of one variable on another one. The word function means doing something that
means it has a unique output as like to the concept of reality. The mathematical term
function’s meaning is coincident to our practical perception because it has only unique
output for single input. The term "function" was introduced by Gottfried Leibniz, in a 1673
and Leonhard Euler introduced the familiar notation "f(x)" for the value of a function.

Definition of Function:
If x and y are two variables related to one another in a such way that each values of x determines
exactly one value of y, then we say that y is a function of x and it is simply written as 𝑦 =
𝑓(𝑥),where x is an independent variable (Argument) and y is a dependent variable. Values of y or
f(x) is called functional value. From real life , concept function works just like a Machine.

For example: 𝑦 = 𝑓(𝑥) = 2𝑥 + 3 is a function of a single variable x where x is an input to the


function or machine and y is an output of the function or machine and in case of function is said as
functional value or value of the function.
Domain of a function: Set of all values of x for which the given function is defined.
Mathematically, D f  x : y  f(x)  R ,  x  R
Range of a function: Set of all values of y corresponding to x such that the given function is defined.

Mathematically, R f  y : y  f(x)  R ,  x  D f 
Types of Functions
Even Function:
A function 𝑓(𝑥) is said to be even function if 𝑓(− 𝑥) = 𝑓(𝑥).
For example: 𝑓(𝑥) = 𝑠𝑖𝑛4 𝑥 is an even function.
Problem: Prove that 𝑓(𝑥) = 𝑠𝑖𝑛2 𝑥 × 𝑐𝑜𝑠 6 𝑥 is an even function.
Proof:
Given function is,𝑓(𝑥) = 𝑠𝑖𝑛2 𝑥 × 𝑐𝑜𝑠 6 𝑥
∴ 𝑓(− 𝑥) = {sin(−𝑥)}2 × {cos(−𝑥)}6
= {𝑠𝑖𝑛3 𝑥} 𝑐𝑜𝑠 6 𝑥
= 𝑠𝑖𝑛3 𝑥 × 𝑐𝑜𝑠 6 𝑥
𝑓(− 𝑥) = 𝑓(𝑥)
Above relation shows that given function 𝑓(𝑥) is an even function. (Proved)

Mohammad Abdul Halim, Lecturer in Mathematics, GED, DIU


Odd Function:
A function 𝑓(𝑥) is said to be odd function if 𝑓(− 𝑥) = − 𝑓(𝑥).
For example: 𝑓(𝑥) = 𝑠𝑖𝑛3 𝑥 is an odd function.
Problem 01: Prove that 𝑓(𝑥) = 𝑠𝑖𝑛3 𝑥 × 𝑐𝑜𝑠 6 𝑥 is an odd function.
Proof:
Given function is,𝑓(𝑥) = 𝑠𝑖𝑛3 𝑥 × 𝑐𝑜𝑠 6 𝑥
∴ 𝑓(− 𝑥) = {sin(−𝑥)}3 × {cos(−𝑥)}6
= {− 𝑠𝑖𝑛3 𝑥} 𝑐𝑜𝑠 6 𝑥
= − 𝑠𝑖𝑛3 𝑥 × 𝑐𝑜𝑠 6 𝑥
𝑓(− 𝑥) = − 𝑓(𝑥)
Above relation shows that given function 𝑓(𝑥) is an odd function. (Proved)
Problem 02: Show that 𝑓(𝑥) = ln(𝑥 + √𝑥 2 + 1) is an odd function of x.
Proof:
Given function is,𝑓(𝑥) = ln(𝑥 + √𝑥 2 + 1)
Now,
𝑓(𝑥) + 𝑓(−𝑥) = ln(𝑥 + √𝑥 2 + 1) + ln (−𝑥 + √(−𝑥)2 + 1)
𝑓(𝑥) + 𝑓(−𝑥) = ln(𝑥 + √𝑥 2 + 1) + ln(−𝑥 + √𝑥 2 + 1)
𝑓(𝑥) + 𝑓(−𝑥) = ln(𝑥 + √𝑥 2 + 1) . (−𝑥 + √𝑥 2 + 1)
𝑓(𝑥) + 𝑓(−𝑥) = ln(√𝑥 2 + 1 + 𝑥) . (√𝑥 2 + 1 − 𝑥)
𝑓(𝑥) + 𝑓(−𝑥) = ln(𝑥 2 + 1 − 𝑥 2 )
𝑓(𝑥) + 𝑓(−𝑥) = ln(1)
𝑓(𝑥) + 𝑓(−𝑥) = 0
∴ 𝑓(𝑥) = −𝑓(−𝑥)
Above relation shows that given function 𝑓(𝑥) is an odd function. (Showed)
Try Yourself:
2
 Prove that 𝑓(𝑥) = 𝑥 2016 𝑠𝑖𝑛2015 𝑥 𝑒 𝑥 is an odd function.
3𝑥 −1
 Show that the function 𝑓(𝑥) = 𝑥 3𝑥 +1 is an even function.
1−𝑥
 Show that the function 𝑓(𝑥) = 𝑙𝑛 1+𝑥 is an odd function.
Homogeneous Function:
A function f (x, y) is said to be homogeneous of degree n if it is possible to expressed as (𝑥, 𝑦) =
𝑦 𝑥
𝑥 𝑛 ∅ (𝑥 ) 𝑜𝑟 𝑓(𝑥, 𝑦) = 𝑦 𝑛 ∅ (𝑦) .
Or
A function is homogeneous of degree n if, when each of its arguments is multiplied by any number
t > 0, the value of the function is multiplied by 𝑡 𝑛 that is 𝑓(𝑡𝑥, 𝑡𝑦) = 𝑡 𝑛 𝑓(𝑥, 𝑦).
Or
A function f is said to be homogeneous if the degree or power of its each term is equal otherwise
Non-homogeneous.
For example: 𝑓(𝑥, 𝑦, 𝑧) = x 2 + y 2 + z 2 + 𝑥𝑦 is a homogeneous function of three variables with
degree 3.
𝑥 4 +𝑦 4
Problem: Prove that 𝑓(𝑥, 𝑦) = 𝑥−𝑦
is a homogeneous function of degree 3.
Proof: Given function is,
𝑥 4 +𝑦 4
𝑓(𝑥, 𝑦) = 𝑥−𝑦
𝑦4
𝑥 4 (1+ 4 )
𝑥
𝑓(𝑥, 𝑦) = 𝑦
𝑥(1− )
𝑥

Mohammad Abdul Halim, Lecturer in Mathematics, GED, DIU


𝑦4
𝑥 3 (1+ 4 )
𝑥
𝑓(𝑥, 𝑦) = 𝑦
(1− )
𝑥
𝑦4
𝑦 𝑦 (1+ 4 )
3 𝑥
𝑓(𝑥, 𝑦) = 𝑥 ∅( ) , where ∅( ) = 𝑦 .
𝑥 𝑥 (1− )
𝑥
Therefore, the given function f(x) is a homogeneous function of degree 3.
2nd process:
Given function is,
𝑥 4 +𝑦 4
𝑓(𝑥, 𝑦) = 𝑥−𝑦
𝑥4
𝑦 4 (1+ 4)
𝑦
𝑓(𝑥, 𝑦) = 𝑥
𝑦(1− )
𝑦
𝑥4
𝑦 3 (1+ 4)
𝑦
𝑓(𝑥, 𝑦) = 𝑥
(1− )
𝑦
𝑥4
𝑥 𝑥 (1+ 4)
3 𝑦
𝑓(𝑥, 𝑦) = 𝑦 ∅ ( ), where ∅( ) = 𝑥 .
𝑦 𝑦 (1− )
𝑦
Therefore, the given function f(x) is a homogeneous function of degree 3.
3rd Process:
Given function is,
𝑥 4 +𝑦 4
𝑓(𝑥, 𝑦) = 𝑥−𝑦
𝑡 4 𝑥 4 +𝑡 4 𝑦4
𝑓(𝑡𝑥, 𝑡𝑦) =
𝑡𝑥−𝑡𝑦
𝑡 4 (𝑥 4 +𝑦 4 )
𝑓(𝑡𝑥, 𝑡𝑦) = 𝑡(𝑥−𝑦)
𝑡 3 (𝑥 4 +𝑦 4 )
𝑓(𝑡𝑥, 𝑡𝑦) = (𝑥−𝑦)
3
𝑓(𝑡𝑥, 𝑡𝑦) = 𝑡 𝑓(𝑥, 𝑦)
Therefore, the given function f(x) is a homogeneous function of degree 3. (Proved)
Try Yourself:
x6 +x6
 Show that the function 𝑓(𝑥) = 𝑥−𝑦
is a Homogeneous function of degree 5 .
x3 +x3
 Find the degree of the function (𝑥) = .
√𝑥−√𝑦
x+y
 If 𝑢 = ,certify that the order of u is -1 .
𝑥 2 +𝑦 2
Periodic function: A function 𝑦 = 𝑓(𝑥) is called a periodic function of period T if it satisfies the
condition 𝑓(𝑥 + 𝑻) = 𝑓(𝑥) where T is least positive real number. It means that the function 𝑓(𝑥)
possess same values after an interval T.
Example: 𝑆𝑖𝑛𝑥 is a periodic function with period 2π
Note:
1. The fundamental period of sinx, cosx, secx, cosecx is 2π and also tanx, cotx is π.
2. The period of a function of the form 𝑦 = 𝐴 𝒕𝒓𝒊𝒈𝒐𝒏𝒐𝒎𝒆𝒕𝒓𝒊𝒄 𝒓𝒂𝒕𝒊𝒐𝒔𝒏 (𝐵𝑥 + 𝐶) is obtained
by dividing the fundamental period of that function by coefficient of x.
𝑓𝑢𝑛𝑑𝑎𝑚𝑒𝑛𝑡𝑎𝑙 𝑝𝑒𝑟𝑖𝑜𝑑 𝑜𝑓 𝑡ℎ𝑒 𝑓𝑢𝑛𝑐𝑡𝑖𝑜𝑛
That is 𝑝𝑒𝑟𝑖𝑜𝑑 = 𝐵×𝑛
.
3. The period of the sum or subtraction of two functions is the LCM (least common multiple)
of the Individual periods.
Problem01: Find the period of the function𝑓(𝑥) = sin4 𝑥 + 𝑐𝑜𝑠 4 𝑥 .
Solution:
Given function is, 𝑓(𝑥) = sin4 𝑥 + 𝑐𝑜𝑠 4 𝑥

Mohammad Abdul Halim, Lecturer in Mathematics, GED, DIU


2𝜋 𝜋
Now the period of the function sin4 𝑥 is = = .
4×1 2
2𝜋 𝜋
Again, the period of the function cos 4 𝑥 is = = .
4×1 2
𝜋 𝜋 𝜋
So, the LCM of 𝑎𝑛𝑑 is .
2 2 2
𝜋
Therefore 𝑓(𝑥) is a periodic function with period . (As desired)
2
Problem02: Find the period of the function𝑓(𝑥) = sin(2𝑥 + 5) + tan(3𝑥 + 7) .
Solution:
Given function is,
𝑓(𝑥) = sin(2𝑥 + 5) + tan(3𝑥 + 7)
Now,
𝑝𝑒𝑟𝑖𝑜𝑑 𝑜𝑓 𝑠𝑖𝑛𝑥 2𝜋
The period of the function of sin(2𝑥 + 5) is = = =𝜋
𝑐𝑜𝑒𝑓𝑓𝑖𝑐𝑖𝑒𝑛𝑡 𝑜𝑓 𝑥 2
Again,
𝑝𝑒𝑟𝑖𝑜𝑑 𝑜𝑓 𝑡𝑎𝑛𝑥 𝜋 𝜋
The period of the function of tan(3𝑥 + 7) is = = =
𝑐𝑜𝑒𝑓𝑓𝑖𝑐𝑖𝑒𝑛𝑡 𝑜𝑓 𝑥 3 3
𝜋 𝜋
Therefore, the period of the function 𝑓(𝑥) is = The LCM of 𝜋 and = . (As desired)
3 3
Try Yorself:
Find the period of the following functions:
1. 𝑓(𝑥) = sin(2𝑥) + cos(3𝑥)
2. 𝑓(𝑥) = tan 5𝑥 + cot3x
𝑥
3. 𝑓(𝑥) = sin(2𝑥) + cos( )
2
Composite Function:
Let 𝑓(𝑥) and 𝑔(𝑥) are two functions then the composition of 𝑓(𝑥) with 𝑔(𝑥) is denoted by 𝑔𝑜𝑓 and the
function
𝑔𝑜𝑓 is defined by (𝑔𝑜𝑓)(𝑥) = 𝑔(𝑓(𝑥)) .
The domain of 𝑔𝑜𝑓 is defined to consist of all x in the domain of 𝑔(𝑥) for which 𝑔(𝑥) is in the domain of
𝑓(𝑥). By the same time composition of 𝑔(𝑥) with 𝑓(𝑥) is denoted by 𝑔𝑜𝑓 is defined by
(𝑓𝑜𝑔)(𝑥) = 𝑓(𝑔(𝑥)) .

𝑔 𝑓

x g(x) f(g(x))

Think like a Machine?

1
Problem: Express the function 𝐹(𝑥) = as a composite function of three functions.
√𝑥+√𝑥
Solution:
1
Given function is 𝐹(𝑥) =
√𝑥+√𝑥
1
Let us consider three functions 𝑔(𝑥) = 𝑥 , 𝑓(𝑥) = 𝑥 + √𝑥 𝑎𝑛𝑑 ℎ(𝑥) = .
√𝑥
Now,
1
ℎ(𝑥) =
√𝑥

Mohammad Abdul Halim, Lecturer in Mathematics, GED, DIU


1
ℎ(𝑔(𝑥)) =
√𝑔(𝑥)
1
ℎ(𝑔(𝑥)) =
√𝑥
1
ℎ(𝑔(𝑓(𝑥))) =
√𝑓(𝑥)
1
ℎ(𝑔(𝑓(𝑥))) =
√𝑥+√𝑥
ℎ(𝑔(𝑓(𝑥))) = 𝐹(𝑥)
ℎ𝑜𝑔𝑜𝑓(𝑥) = 𝐹(𝑥)
Therefore, the given function is expressed as a composition of three functions. (Ans)
Try yourself:
1. Express the function as a composite function of three functions.

Inverse Function: If 𝑦 = 𝑓(𝑥) is the function of x then the inverse of the function is 𝑓 −1 (𝑦) = 𝑥.
It exists if 𝑓 −1 (𝑓(𝑥)) = 𝑥.
Problem: Find the inverse of the function f ( x)  ln( x  x 2  1) .
Solution: Given function is , f ( x)  ln( x  x 2  1)
Let y  ln( x  x 2  1)
e y  x  x2  1
e y  x  x2  1

e   
2 2
y
x  x2  1
e  2 xe  x  x  1
2y y 2 2

e 2 y  2 xe y  1
2 xe y  e 2 y  1
e2 y  1
x
2e y
1
f 1 (y)  e y  e  y
2
 
Replacing x by y we get,

f 1 ( x) 
2

1 x x
e e 
Therefore the inverse of the given function is f
1
( x) 
2

1 x x

e e .
Try Yorself:
1 x 
1. Find the inverse of the function h ( x )  ln .
1 x 
 1 x 
2. Find the inverse of the function h ( x )  ln  .
 1 x 
3. Find 𝑓 −1 (𝑥) for the function f ( x)  ln( x  x 2  1) .
4. Find 𝑓 −1 (𝑥) for the function f ( x)  ln( x  x 2  1) .

Mohammad Abdul Halim, Lecturer in Mathematics, GED, DIU


Chapter 02 : Domain and Range of Functions
We will discuss in this section about the domain and range of linear functions, quadratic functions,
square root functions and rational functions.

Linear function:

Problem : Find the domain and range of the function f ( x)  2 x  5 .


Solution:
Domain:Given function is,
f ( x)  2 x  5  R if and if x  R .
That means x takes all real values from the real number set R and produces real outputs.
Therefore the domain of this function is the real number set.
So, the domain of the given function is, D f  R .
Note: For calculating the domain of a function it is must to take the restriction on x.
Range:
Let y  f ( x )  2 x  5 .
y  2x  5
2x  y  5
y 5
x R
2
That means y takes all real values from the real number set R and produces real outputs.
Therefore the range of this function is the real number set.
So, the range of the given function is, R f  R .
Note: For calculating the range of a function it is must to take the restriction on y.
Try yourself:
Find the domain and range of the following functions
1. f ( x)  3x  5 2. f ( x)  2 x  5 3. f ( x)  ax  b

Quadratic function:

Problem : Find the domain and range of the function f ( x)  x 2  3x  2 .


Solution:
Domain:Given function is,
 
f ( x)  x 2  3 x  2  R if and if x  R .
That means x takes all real values from the real number set R and make sense.
Therefore the domain of this function is the real number set.
So, the domain of the given function is, D f  R .
Range:Given function is,
y  f ( x )  x 2  3x  2 [Say]
y  x  3x  2
2

x 2  3x   2  y   0
In the above equation the value of x will be real if and only if

Mohammad Abdul Halim, Lecturer in Mathematics, GED, DIU


32  4.1.(2  y)  0
9  4(2  y )  0
9 8 4y  0
1 4 y  0
4 y  1
1
y
4
1
Therefore the range of the given function is R f  [ , ) . (Ans)
4
Try Yourself:
Find the domain and range of the following quadratic functions
1. f ( x)  x 2  5 x  6 2. f ( x)  x 2  4 x  3 3. y   x  2   3
2

Square root function:


Problem: Find the domain and range of the function f ( x )  2 x  5 .
Solution:
Domain: Given function is,
f ( x)  2 x  5  R , iff 2 x  5  0
2x   5
5
x  
2
5
Therefore the domain of the given function is D f  {x : x   } .
2
5
D f  [ , )
2
Range: Suppose y  f ( x)  2 x  5 , y  0
Here y2  2x  5 [Squaring both sides]
2x  5  y 2

2x  y2  5
y2  5
x  R for all y  R .
2
Now, the range of the given function is R f  {y : y  0}  {y : y  R}
R f  [0, )  (  , )
R f  [0, ) (Ans)
Note: The range, most of the positive square root function is R f  [0, ) .
Try Yourself:
Find the domain and range of the functions:
1. f ( x)  x  6  5 2. f ( x)  2 x  6 3. f ( x)  1  2 x  6

Mohammad Abdul Halim, Lecturer in Mathematics, GED, DIU


Rational function:
2x 1
Problem: Find the domain and range of the function f ( x)  .
x5
Solution:
Domain: Given Rational function is,
2x 1
f ( x)   R iff x  5  0
x 5
 x5
That means given function f ( x ) produces real output for all real values of x accept x  5 .
Therefore the domain of the given function f ( x ) is,

D f  R  5
2x 1
For Range: Suppose y  f ( x ) 
x5
2x 1
 y
x5
xy  5 y  2 x  1
xy  2 x  5 y  1
x  y  2  5 y  1
5y 1
x  R if y  2  0  y  2
y2
It shows x produces real values for all values of y accept y  2 .
Therefore the domain of the given function f ( x ) is,
R f  R  2
Try Yourself:
Find the domain and range of the functions:
x2  4 x 2  16
1. f ( x)  2. f ( x)  3. f ( x)  1  2 x  6
x2 x4

Mohammad Abdul Halim, Lecturer in Mathematics, GED, DIU


Chapter 03 : Graph of Functions
If 𝑓: 𝐴 → 𝐵 denotes a function, then the graph of the function 𝑓(𝑥) is the set of all ordered pairs
(𝑥, 𝑓(𝑥)) for all values of x in the domain A.
∴ Graph of 𝑓(𝑥) = {(𝑥, 𝑦): 𝑥𝜖𝐴, 𝑦 = 𝑓(𝑥)𝜖𝐵}
Therefore Graph is the geometrical/Pictorial representation of a function or visualization of a
function.
Human being is not able to observe its face without mirror as like this we don’t observe equation
without sketching/ graphing it geometrically.
Graph of Some elementary functions

1. Y Y

m>0 m<0
X X
O O

Graph of 𝑦 = 𝑚𝑥 + 𝑐 Graph of 𝑦 = 𝑚𝑥 + 𝑐

2. 3.
Y Y

O
X X
O

Graph of 𝑦 = 𝑚𝑥 Graph of 𝑦 = 𝑥

Mohammad Abdul Halim, Lecturer in Mathematics, GED, DIU


4. Y 5. Y

O
X X
O

Graph of 𝑦 = − 𝑥 Graph of y = |x|

6. Y Y

X X
O O

Graph of y = x 2 Graph of y = − x 2

Y Y

O X X
O

Graph of x = y 2
Note: when power of the variable increases Graph of x = − y 2
then graph will be wider

Mohammad Abdul Halim, Lecturer in Mathematics, GED, DIU


Y Y

O
X X
O

Graph of y = √x Graph of y = − √x

Y Y

X X
O O

Graph of y = √− x Graph of y = − √− x

Y
Y

X
O X
O

1 1
Graph of y = Graph of y =
x x2

Mohammad Abdul Halim, Lecturer in Mathematics, GED, DIU


Y Y

X X
O O

Graph of y = x 3 3
Graph of y = √x

Y Y

(0,1)
(0,1)
X X
O O

Graph of y = ex Graph of y = e− x

Y Y

(1,0) (0,1)
X X
O O

Graph of y = ln|x| Graph of y = ax , a > 1

Mohammad Abdul Halim, Lecturer in Mathematics, GED, DIU


Y Y

(0,1)
(1,0)
X X
O O

Graph of y = a− x Graph of y = log a |x|, a > 1

Y Y

-a O a
X X
-a O a

Graph of y = √a2 − x 2 Graph of y = −√a2 − x 2

Y Y
b

-a a -a a
X X
O O
-b

x2 y2
Graph of x 2 + y 2 = a2 Graph of a2
+ b2 = 1

Mohammad Abdul Halim, Lecturer in Mathematics, GED, DIU


Y

X
O

x2 y2
Graph of 2
− =1 Graph of y = sinx
a b2

Graph of y = cosx

Transformation of functions
Transformation of a function is any kind of change in the function such as move or resize the graphs
of functions. There are two types of transformation of the functions such as,
1. Translation/Shifting: Any kind of shifting of the graph of a function is called translation of
the function that means changing the location of the graph without changing its size and shape
is called translation.
2. Scaling: Scaling of a graph of a function is a transformation in which the size and shape of
the graph is changed.
Translation:
Horizontal translation: Function: g(x) = f(x + c)

For c > 0 the graph is translated c units to the left.

Mohammad Abdul Halim, Lecturer in Mathematics, GED, DIU


For c < 0 the graph is translated c units to the right.

Vertical Translation: Function: g(x) = f(x) + c

For c > 0 the graph is translated c units upward.

For c < 0 the graph is translated c units downward.


Scaling: Function: g(x) = cf(x)

For |c| > 1 (integer) the graph is compressed.

For |c| < 1 (integer) the graph is stretched.

Mohammad Abdul Halim, Lecturer in Mathematics, GED, DIU


Problem01: Sketch the graph of the function f ( x)  x 2  6 x  10 .

Solution:
The equation of the given function is,
y  f ( x)  x 2  6 x  10 [ Say]
Completing the given equation in a square form it becomes as
y  x 2  6 x  10
y  x 2  2.x.3  32  32  10
y   x  3  9  10
2

y   x  3  1
2

The graph of the standard function y  x 2 is as follows

Translating or shifting the above graph 3 units to the left, we get the graph of the
function y   x  3
2
.

Translating or shifting the above graph 1 units upward, we get the graph of the
function y   x  3  1 .
2

(Desired Graph)
Try Yourself: Sketch the graph of the following functions
1. f ( x)  x 2  4 x  10 2. f ( x)  2 x 2  5 x  10 3. f ( x)  x 2  4 x  5

Mohammad Abdul Halim, Lecturer in Mathematics, GED, DIU


Problem02: Sketch the graph of the function f ( x)  x2 5.
Solution:
The equation of the given function is,
y  f ( x)  x  2  5 [ Say ]
The graph of the standard positive square root function y  x is as follows

Translating or shifting the above graph 2 units to the right, we get the graph of the
function y  x2.

Translating or shifting the above graph 5 units upward, we get the graph of the
function y  x2 5 .

(Desired Graph)

Try Yourself: Sketch the graph of the following functions

1. f ( x)  x  2 2. f ( x )  2x  5 3. f ( x)  2  x  5

Problem03: Sketch the graph of the function f ( x)  2  x  2 .

Solution:
The equation of the given function is,
y  f ( x)  2  x  2 [Say]
The graph of the standard absolute value function y  x is as follows

Mohammad Abdul Halim, Lecturer in Mathematics, GED, DIU


Therefore the graph of the standard absolute value function
y   x is as follows

Translating or shifting the above graph 2 units to the left, we get the graph of the
function y   x  2 .

Translating or shifting the above graph 2 units upward, we get the graph of the function
y   x  2  2 or y  2  x  2 .

(Desired Graph)

Try Yourself: Sketch the graph of the following functions

1. f ( x)  x  2  2 2. f ( x)  x  2  3 3. f ( x)  1  x  3

Piecewise function:
A piecewise-defined function (also called a piecewise function or a hybrid
function) is a function which is defined by multiple sub-functions, each sub-function
applying to a certain interval of the main function's domain (a sub-domain).
For example: The following function is the piecewise function

Mohammad Abdul Halim, Lecturer in Mathematics, GED, DIU


 f1 (x) , xa

y  f ( x)   f 2 (x) , a  x b
 f (x) , xb
 3
0 , x  1

Problem04: Sketch the graph of the function f ( x)   1  x 2 , 1  x  1 .
x , x 1


Solution:
Given function is
0 , x  1

y  f ( x)   1  x 2 , 1  x  1 [ say]
x , x 1

In the interval x  1 or (, 1] , the graph of the function f ( x ) is horizontal line
y  0.

In the interval 1  x  1 or ( 1,1) , the graph of the function f ( x ) is an upper semi-

circle y  1  x 2 of radius 1 units and Centre at origin.

Again,

In the interval x  1 or [1, ) , the graph of the function f ( x ) is a straight line y  x of


radius 1 units and Centre at origin.

Mohammad Abdul Halim, Lecturer in Mathematics, GED, DIU


Therefore the graph of the given function is as follows:

(Desired Graph)


 x2 1 ,x 0

Problem05: Sketch the graph of the function f ( x)   x ,0  x 1 .
1
 , x 1
x
Solution:
Given function is,

 x2 1 ,x 0

y  f ( x)   x ,0  x 1 [ say ]
1
 , x 1
x
We know the graph of the function y  x 2 , x  0 is as follows

Mohammad Abdul Halim, Lecturer in Mathematics, GED, DIU


So, we get the graph of the function y  x 2  1 , x  0 if we translate the above graph
1 unit down.

The graph of the function y  x , 0  x  1 is as follows

1
And, the graph of the function y  , x  1 is as follows:
x

Finally the graph of the given function is as follows

(1, 0)

(Desired Graph)
2 x  1 ,x 0

Problem06: Sketch the graph of the function f ( x)   1 ,0  x 1 .
 2x 1 , x 1

Solution:
Given function is,

Mohammad Abdul Halim, Lecturer in Mathematics, GED, DIU


2 x  1 ,x0

y  f ( x)   1 ,0  x 1 [ Say]
 2x 1 , x 1

In the interval x  0 or (, 0) , the graph of the function f ( x ) is a straight line


y  2 x  1 .

In the interval 0  x  1 or [0,1) , the graph of the function f ( x ) is a straight line y  1 .

Again, In the interval x  1 or [1, ) , the graph of the function f ( x ) is a straight line
y  2x 1 .

Finally the graph of the given function is as follows:

(Desired Graph)

Mohammad Abdul Halim, Lecturer in Mathematics, GED, DIU


Try Yourself: Sketch the graph of the following piecewise functions:

  0 , x 1
 x2 ,x 0  x2  1 ,x 0
 

1. f ( x)   x ,0  x 1 2. f ( x)   x ,0  x 1 3. f ( x)  1  x , 1  x  0
1 1 1  x ,0  x 1
 , x 1  , x 1 
x x

Modulus/absolute function:
x
The modulus or absolute value of x is denoted by the symbol and is defined as follows
 x , x0
x 
 x , x  0
Geometrically the modulus or absolute value of a number represents the distance of that number from
the origin. The absolute value of x is always positive or zero.
A function together with modulus or absolute value sign is called modulus function.
For example: The function f ( x)  5 x  3  2 x  2 is an absolute value function or Modulus
function.
What is breaking point of a function?
Breaking point of a function is a point at which the function changes.
For example: The function f ( x)  5 x  3  2 x  2 has two breaking points are x  3& x  2 .
Procedure of Graphing Absolute value function:
1. At first convert the modulus function into piecewise function according to its number of
breaking points.
2. After that sketch the graph as piecewise function.

Problem07: Sketch the graph of the function f ( x)  x  x  1 .

Solution:
Given absolute value function is,
y  f ( x)  x  x  1 [Say]
For breaking points x  0 and x  1  0  x  1 .

0 1
There are two breaking points in this mathematical problem such as x  0 & x  1 and those points
divide real number line into three intervals like as   ,0 , [0,1) and [1, ) .Therefore we define
this absolute value function section-ally by three parts.
Now,
y  f ( x)  x  x  1
y  f ( x)    x      x  1 

Mohammad Abdul Halim, Lecturer in Mathematics, GED, DIU


  x      x  1  ,x 0

y  f ( x)    x      x  1  ,0  x 1

  x      x  1  , x 1
 x   x  1 ,x 0

y  f ( x)   x   x  1 , 0  x  1

 x   x  1 , x 1
2 x  1 ,x 0

y  f ( x)   1 ,0  x 1
 2x 1 , x  1

Graph
In the interval x  0 or (, 0) , the graph of the function f ( x ) is a straight line y  2 x  1 .

In the interval 0  x  1 or [0,1) , the graph of the function f ( x ) is a straight line y  1 .

Again, In the interval x  1 or [1,  ) , the graph of the function f ( x ) is a straight line y  2 x  1 .

Mohammad Abdul Halim, Lecturer in Mathematics, GED, DIU


Finally the graph of the given function is as follows:

(Desired Graph)

Problem08: Sketch the graph of the function f ( x)  x  1  x  x  1 .

Solution:
Given absolute value function is,
y  f ( x)  x  1  x  x  1 [Say]
For breaking points x  1  0  x  1 and x  0 an also x  1  0  x  1

-1 0 1
There are three breaking points in this mathematical problem such as x  1 , x  0 & x  1 and
those points divide real number line into four intervals like as   ,  1 , [ 1, 0) , [0,1) and [1, )
.Therefore we define this absolute value function section-ally by four parts.
Now,
y  f ( x)  x  1  x  x  1
y  f ( x)     x  1     x      x  1 

   x  1     x      x  1  , x  1

   x  1     x      x  1  , 1  x  0
y  f ( x)  
   x  1     x      x  1  ,0  x 1

   x  1     x      x  1  , x 1

  x  1  x   x  1 , x  1

  x  1  x   x  1 , 1  x  0
y  f ( x)  
  x  1  x   x  1 ,0  x 1
 x  1  x  x  1
     , x 1

Mohammad Abdul Halim, Lecturer in Mathematics, GED, DIU


3 x , x  1
 x  2 , 1  x  0

y  f ( x)  
 x2 ,0  x 1
 3 x , x 1

Graph
In the interval x  1 or [1, 0) , the graph of the function f ( x ) is a straight line y  3 x .

In the interval 1  x  0 or [1, 0) , the graph of the function f ( x ) is a straight line y   x  2 .

In the interval 0  x  1 or [0,1) , the graph of the function f ( x ) is a straight line y  x  2 .

In the interval x  1 or [1,  ) , the graph of the function f ( x ) is a straight line y  3x .

Mohammad Abdul Halim, Lecturer in Mathematics, GED, DIU


Finally the graph of the given function is as follows:

Try Yourself: Sketch the graph of the following absolute value functions:

1. f ( x)  x  x  1
2. f ( x)  x  1  x  1
3. f ( x)  x  1  x  2

Chapter 04 : Differential Coefficients


𝑑𝑦
Differential Coefficient: Differential Coefficient 𝑑𝑥 is the changing rate of y in terms of x.
Formulae: If 𝑢 = 𝑢(𝑥) and 𝑣 = 𝑣(𝑥) then
d dv du
1. (uv)  u v (Product Rule)
dx dx dx
d du dv dw
2. (uvw)  vw  wu  uv (Product Rule)
dx dx dx dx
du dv
v u
d u dx dx (Rational Function)
3.  
dx  v  v 2

4.
d v
dx
 
u  uv
d
dx
 v ln u  (Power Function)
5.
d
dx
 x 
1
2 x
for y  ln  sec x  tan x  .
𝑑𝑦
Problem01: Find
𝑑𝑥
Solution: Given that
y  ln  sec x  tan x 

  ln  sec x  tan x  


dy d 1 d
.  sec x  tan x  

sec x tan x  sec 2 x 
dx dx sec x  tan x dx sec x  tan x

Mohammad Abdul Halim, Lecturer in Mathematics, GED, DIU


sec x  tan x  sec x 
  sec x (As desired).
sec x  tan x
𝑑𝑦 1  cos x
Problem02: Find 𝑑𝑥 for y  .
1  cos x
Solution: Given that
1  cos x
y
1  cos x
d d
dy d  1  cos x  1  cos x   1  cos x   1  cos x   1  cos x 
   dx dx

dx dx  1  cos x  1  cos x 
2


1  cos x    0  sin x   1  cos x    0  sin x 
1  cos x 
2


1  cos x    sin x   1  cos x    sin x   sin x (1  cos x  1  cos x)
1  cos x  1  cos x 
2 2

x x
2.2sin cos
2.sin x 2 2  tan x  sec 2 x
 
1  cos x 
2
x 2 2
4 cos 4
2

2nd Process:
 x
2sin 2 
dy d  1  cos x  d  2  d  tan 2 x x 2 x 1 x 2 x
         2  tan  sec   tan  sec
dx dx  1  cos x  dx  2cos 2 x  dx  2 2 2 2 2 2
 2
(As desired).
𝑑𝑦 cos x  sin x
Problem03: Find for y  .
𝑑𝑥
1  sin 2 x
Solution: Given that,
cos x  sin x
y
1  sin 2 x
cos x  sin x

sin x  cos 2 x  2sin x cos x
2

cos x  sin x

 cos x  sin x 
2

cos x  sin x
 1
cos x  sin x
 y 1
dy d
Therefore,  (1)  0. (As desired).
dx dx

Mohammad Abdul Halim, Lecturer in Mathematics, GED, DIU


𝑑𝑦 1  x2 1
Problem04: Find 𝑑𝑥 for y  tan 1 .
x
1  x2 1
Solution: Given that, y  tan 1
x
Letting x  tan  ,we get
1  tan 2   1 sec2 x  1 sec x  1
y  tan 1  tan 1  tan 1
tan  tan  tan 
1 1  cos 
1
 1  cos  cos  
 tan 1 cos   tan 1 cos   tan 1   
sin  sin   cos  sin  
cos  cos 
  
 2sin 2

 tan 1 
1  cos   1 2 
  tan   
 sin    2sin cos 
 2 2
  1
 tan 1 tan    tan 1 x
2 2 2
dy d  1  1 1 1
Therefore,    tan 1 x     (As desired).
dx dx  2  2 1 x 2
2 1  x2
𝑑𝑦
Problem05: Find 𝑑𝑥 for 𝑦 = √𝑥 𝑒 𝑥 sec 𝑥.
Solution: Given that,
𝑦 = √𝑥 𝑒 𝑥 sec 𝑥
𝑑𝑦 𝑑
= (√𝑥 𝑒𝑥 sec 𝑥)
𝑑𝑥 𝑑𝑥
𝑑 𝑑
= √𝑥 𝑑𝑥 (𝑒 𝑥 sec 𝑥) + 𝑒 𝑥 sec 𝑥 𝑑𝑥 √𝑥
𝑑 𝑑 𝑒𝑥 sec 𝑥
= √𝑥 [𝑒𝑥 𝑑𝑥 (sec 𝑥) + sec 𝑥 𝑑𝑥 𝑒𝑥 ] + 2√𝑥
𝑥 𝑥 𝑒𝑥 sec 𝑥
= √𝑥[𝑒 sec 𝑥 tan 𝑥 + 𝑒 sec 𝑥] + 2√𝑥
𝑒𝑥 sec 𝑥
= √𝑥𝑒𝑥 sec 𝑥 [tan 𝑥 + 1] + 2√𝑥
1
= √𝑥𝑒𝑥 sec 𝑥 [tan 𝑥 + 1 + 2𝑥] (As desired)
…….∞
𝑑𝑦 𝑥𝑥
Problem06: Find 𝑑𝑥 for 𝑦 = 𝑥 𝑥 .
…….∞
𝑥𝑥
Solution: Given that, 𝑦 = 𝑥 𝑥 = 𝑥𝑦
𝑑𝑦 𝑑 𝑑 1 𝑑𝑦
= 𝑑𝑥 (𝑥 𝑦 ) = 𝑥 𝑦 𝑑𝑥 ( 𝑦 ln 𝑥) = 𝑥 𝑦 [𝑦. 𝑥 + ln 𝑥 𝑑𝑥 ]
𝑑𝑥
𝑑𝑦 𝑑𝑦 𝑦 𝑑𝑦 𝑦
⇒ 𝑑𝑥 − 𝑥 𝑦 ln 𝑥 𝑑𝑥 = 𝑥 𝑥 𝑦 ⇒ 𝑑𝑥 (1 − 𝑥 𝑦 ln 𝑥) = 𝑥 𝑥 𝑦
𝑑𝑦 𝑦𝑥 𝑦 𝑦2
⇒ 𝑑𝑥 = 𝑥(1−𝑥 𝑦 ln 𝑥) = 𝑥(1−𝑦 ln 𝑥) ; [𝑦 = 𝑥 𝑦 ] (As desired)
𝑑𝑦
Problem07: Find 𝑑𝑥 for 𝑦 = (tan 𝑥)cot 𝑥 + (cot 𝑥)tan 𝑥 .
𝑑𝑦 𝑑 𝑑
= (tan 𝑥)cot 𝑥 + 𝑑𝑥 (cot 𝑥)tan 𝑥
𝑑𝑥 𝑑𝑥

Mohammad Abdul Halim, Lecturer in Mathematics, GED, DIU


𝑑 𝑑
= (tan 𝑥)cot 𝑥 𝑑𝑥 {cot 𝑥 ln(tan 𝑥) + (cot 𝑥)tan 𝑥 𝑑𝑥 {tan 𝑥 ln(cot 𝑥)}
𝑑 𝑑 𝑑
= (tan 𝑥)cot 𝑥 {cot 𝑥 𝑑𝑥
ln(tan 𝑥) + ln(tan 𝑥) 𝑑𝑥 cot 𝑥} + (cot 𝑥)tan 𝑥 {tan 𝑥 𝑑𝑥 ln(cot 𝑥) +
𝑑
ln(cot 𝑥) 𝑑𝑥 tan 𝑥}
cot 𝑥 sec2 𝑥 −𝑐𝑜𝑠𝑒𝑐2 𝑥
= (tan 𝑥)cot 𝑥 { tan 𝑥
+ ln (tan 𝑥) (−𝑐𝑜𝑠𝑒𝑐 2 𝑥)} + (cot 𝑥)tan 𝑥 {tan 𝑥
cot 𝑥
+
ln(cot 𝑥) sec 2 𝑥}
= (tan 𝑥)cot 𝑥 {𝑐𝑜𝑠𝑒𝑐2 𝑥 + ln(tan 𝑥) (−𝑐𝑜𝑠𝑒𝑐 2 𝑥)} + (cot 𝑥)tan 𝑥 {− sec2 𝑥 + ln(cot 𝑥) sec 2 𝑥}
= (tan 𝑥)cot 𝑥 {1 − ln(tan 𝑥)}𝑐𝑜𝑠𝑒𝑐2 𝑥 − (cot 𝑥)tan 𝑥 {1 − ln(cot 𝑥)} sec 2 𝑥 (As desired)
𝑑𝑦 𝑦 𝑥
Problem08: Find for (cos 𝑥) + (sin 𝑦) = 0 .
𝑑𝑥
Solution: Given Equation,
(cos 𝑥)𝑦 + (sin 𝑦)𝑥 = 0
𝑑 𝑑
⇒ 𝑑𝑥 (cos 𝑥)𝑦 + 𝑑𝑥 (sin 𝑦)𝑥 = 0
𝑑 𝑑
⇒ (cos 𝑥)𝑦 𝑑𝑥 [𝑦 ln cos 𝑥] + (sin 𝑦)𝑥 𝑑𝑥 [𝑥 ln sin 𝑦] = 0
𝑑 𝑑𝑦 𝑑 𝑑
⇒ (cos 𝑥)𝑦 [𝑦 ln cos 𝑥 + ln cos 𝑥 ]+ (sin 𝑦)𝑥 [𝑥 𝑑𝑥 ln sin 𝑦 + ln sin 𝑦 . 1] = 0
𝑑𝑥 𝑑𝑥 𝑑𝑥
𝑑𝑦 𝑑𝑦
⇒ (cos 𝑥)𝑦 [𝑦(− tan 𝑥) + ln cos 𝑥 ] + (sin 𝑦)𝑥 [𝑥 cot 𝑦 + ln sin 𝑦 . 1] = 0
𝑑𝑥 𝑑𝑥
𝑑𝑦 𝑑𝑦
⇒ (cos 𝑥)𝑦 [ln cos 𝑥 𝑑𝑥 ] + (sin 𝑦)𝑥 [𝑥 cot 𝑦 𝑑𝑥 ] = (𝑦 tan 𝑥)(cos 𝑥)𝑦 − (sin 𝑦)𝑥 ln sin 𝑦
𝑑𝑦
⇒ [(cos 𝑥)𝑦 ln cos 𝑥 + (sin 𝑦)𝑥 𝑥 cot 𝑦] 𝑑𝑥 = (𝑦 tan 𝑥)(cos 𝑥)𝑦 − (sin 𝑦)𝑥 ln sin 𝑦
𝑑𝑦 (𝑦 tan 𝑥)(cos 𝑥)𝑦 −(sin 𝑦)𝑥 ln sin 𝑦

𝑑𝑥
= [(cos 𝑥)𝑦 ln cos 𝑥+(sin 𝑦)𝑥 𝑥 cot 𝑦]
(As desired)
𝑑𝑦
Try yourself: Find 𝑑𝑥 for the followings

1. y  ln  x a  x b  2. y 
1  sin x
1  sin x
3. 𝑦 = √𝑥 105𝑥 cosec 𝑥.

 1 x  1  x2  1 6. y  tan 1
x
4. y  sin  2 tan 1  5. y  tan 1
 1  x  x 1  x2  1

7. ) 𝑥 𝑦 + 𝑦 𝑥 = 𝑎𝑏 8. y  xcos x  cos1 x tan x 9. y   sin x 
cos x
 x ln(sin x)

Derivative of a parametric Equations


When two variables 𝑥 𝑎𝑛𝑑 𝑦 are expressed as another variable 𝜃 then 𝜃 is called a parameter of x
and y. A parametric equation is an equation in which a parameter exists.
𝑑𝑦 𝑑𝑦 𝑑𝜃 𝑑𝑦 𝑑𝑥
Formula: 𝑥 = 𝑓(𝜃) and 𝑦 = 𝑔(𝜃) then 𝑑𝑥
= 𝑑𝜃 ∙ 𝑑𝑥 = 𝑑𝜃⁄𝑑𝜃
𝑑𝑦
Problem 09: Find 𝑑𝑥 for x  a (  cos  ) & y  b(sin   cos  ) .
Solution: Given parametric Equations are,
x  a (  cos  ) & y  b(sin   cos  )
dx d
Now,  a (  cos  )
d dx

Mohammad Abdul Halim, Lecturer in Mathematics, GED, DIU


d
a (  cos  )  a 1  sin  
dx
dy d d
Again,  b(sin   cos  )  b (sin   cos  )  b(cos   sin  )
d d d
dy dy d 1 b(cos   sin  )
Therefore,  .  b(cos   sin  )   (As desired)
dx d dx a 1  sin   a 1  sin  
dy
Try Yourself: Find for the followings parametric equations,
dx
1. x  a (sin   cos  ) & y  b cos  2. x  a(sint  cost) & y  b  cost  sint 

Derivative of a functions with respect to another functions

1 1  x
2
2x 2x
Formula: 2 tan 1 x  sin 1  cos  tan 1
1 x 2
1 x 2
1  x2
1  x2 1 2x
Problem: Differentiate tan 1 with respect to tan 1 .
x 1  x2
1  x2 1
Solution: Let, z  tan 1
x
Letting x  tan  ,we get
1  tan 2   1 sec2 x  1 sec x  1
z  tan 1  tan 1  tan 1
tan  tan  tan 
1 1  cos 
1
 1  cos  cos  
 tan 1 cos   tan 1 cos   tan 1   
sin  sin   cos  sin  
cos  cos 
 2 
 2sin 
1  1  cos   1 2
 tan    tan 
 sin   
 2sin cos 

 2 2
  1
 tan 1 tan    tan 1 x
2 2 2
dz d  1  1 1 1
Therefore,    tan 1 x    
dx dx  2  2 1 x 2 1  x 2 
2

2x
Again, y  tan 1  2 tan 1 x
1 x 2

dy d

dx dx

2 tan 1 x 
2
1  x2

dz dz dx 1 1  x2
Now,  .   1 (As desired)
dy dx dy 2 1  x 2  2 

Mohammad Abdul Halim, Lecturer in Mathematics, GED, DIU


Try Yourself:
1  x2 1 2x
1. Differentiate tan 1 with respect to sin 1 .
x 1  x2
2x 2x
2. Differentiate tan 1 with respect to sin 1 .
1 x 2
1  x2

Mohammad Abdul Halim, Lecturer in Mathematics, GED, DIU


Chapter 05 : Successive Differentiation
𝑑𝑦
 If 𝑦 = 𝑓(𝑥) then the first order differential coefficient with respect to 𝑥 is denoted by 𝑑𝑥
,
𝑑𝑓(𝑥)
𝑦1 , 𝑦 ′ , 𝑦 (1) , 𝐷𝑦, , 𝑓 ′ (𝑥), 𝑓 (1) (𝑥), 𝑓𝑥 (𝑥), 𝐷𝑓(𝑥), 𝐷𝑥 𝑓(𝑥) etc.
𝑑𝑥
 If 𝑦 = 𝑓(𝑥) then the second order differential coefficient with respect to 𝑥 is denoted by
𝑑2 𝑦 𝑑 2 𝑓(𝑥)
𝑑𝑥 2
, 𝑦2 , 𝑦 ′′ , 𝑦 (2) , 𝐷 2 𝑦, 𝑑𝑥 2
, 𝑓 ′′ (𝑥), 𝑓 (2) , 𝑓𝑥𝑥 (𝑥), 𝐷 2 𝑓(𝑥), 𝐷𝑥𝑥 𝑓(𝑥) etc.
 If 𝑦 = 𝑓(𝑥) then the n-th order differential coefficient with respect to 𝑥 is denoted by
𝑑𝑛𝑦 𝑑 𝑛 𝑓(𝑥)
, 𝑦𝑛 , 𝑦 (𝑛) , 𝐷 𝑛 𝑦, , 𝑓 (𝑛) (𝑥), 𝐷 𝑛 𝑓(𝑥) etc.
𝑑𝑥 𝑛 𝑑𝑥 𝑛
Mathematical Problems
𝒏
Problem 01: If 𝒚 = 𝒙 then 𝒚𝒏 =?
Solution: Given 𝑦 = 𝑥 𝑛
 y1  nx n 1
 y2  n  n  1 xn2
 y3  n  n  1 n  2 xn3
Proceeding in this way we get,
yn  n  n  1 n  2  n   n  1 x nn

 n  n  1 n  2  3.2.1
 n!
(As desired)
𝟐𝒏
Problem 02: If 𝒚 = 𝒙 then 𝒚𝒏 =?
Solution: Given 𝑦 = 𝑥 2𝑛
⇒ 𝑦1 = 2𝑛 𝑥 2𝑛−1
⇒ 𝑦2 = 2𝑛(2𝑛 − 1)𝑥 2𝑛−2
⇒ 𝑦3 = 2𝑛(2𝑛 − 1)(2𝑛 − 2)𝑥 2𝑛−3
… … … … … …… … … …
⇒ 𝑦𝑛−1 = 2𝑛(2𝑛 − 1)(2𝑛 − 2) … … (2𝑛 − 𝑛 + 2)𝑥 2𝑛−𝑛+1
= 2𝑛(2𝑛 − 1)(2𝑛 − 2) … … (2𝑛 − 𝑛 + 2)𝑥 𝑛+1
⇒ 𝑦(𝑛−1)+1 = 2𝑛(2𝑛 − 1)(2𝑛 − 2) … … (𝑛 + 2)(𝑛 + 1)𝑥 𝑛+1−1
⇒ 𝑦𝑛 = 2𝑛(2𝑛 − 1)(2𝑛 − 2) … … (𝑛 + 2)(𝑛 + 1)𝑥 𝑛
2𝑛(2𝑛−1)(2𝑛−2)……(𝑛+2)(𝑛+1)𝑛(𝑛−1)(𝑛−2)…3.2.1 𝑛
⇒ 𝑦𝑛 = 𝑛(𝑛−1)(𝑛−2)…3.2.1
𝑥
(2𝑛)! 𝑛
⇒ 𝑦𝑛 = 𝑛!
𝑥
= {2𝑛(2𝑛 − 2) … … 6.4.2}{(2𝑛 − 1)(2𝑛 − 3) … … 5.3.1}𝑥 𝑛
= 2𝑛 {𝑛(𝑛 − 1) … … 3.2.1}{(2𝑛 − 1)(2𝑛 − 3) … … 5.3.1}𝑥 𝑛
= 2𝑛 𝑛! {(2𝑛 − 1)(2𝑛 − 3) … … 5.3.1}𝑥 𝑛 (As desired)

Mohammad Abdul Halim, Lecturer in Mathematics, GED, DIU


Problem 03: If 𝒚 = 𝐬𝐢𝐧(𝒂𝒙 + 𝒃) then 𝒚𝒏 =?
Solution: Given 𝑦 = sin(𝑎𝑥 + 𝑏)
𝜋
⇒ 𝑦1 = 𝑎 cos(𝑎𝑥 + 𝑏) = 𝑎 sin { 2 + (𝑎𝑥 + 𝑏)}
𝜋 𝜋 𝜋 𝜋
⇒ 𝑦2 = 𝑎2 cos { 2 + (𝑎𝑥 + 𝑏)} = 𝑎2 sin { 2 + 2 + (𝑎𝑥 + 𝑏)} = 𝑎2 sin {2 2 + (𝑎𝑥 + 𝑏)}
𝜋 𝜋
⇒ 𝑦3 = 𝑎3 cos {2 + (𝑎𝑥 + 𝑏)} = 𝑎3 sin {3 + (𝑎𝑥 + 𝑏)}
2 2
… … … … … …… … … …
1 𝜋 𝜋
⇒ 𝑦𝑛 = 𝑎𝑛 cos {(𝑛 − ) + (𝑎𝑥 + 𝑏)} = 𝑎𝑛 sin {𝑛 + (𝑎𝑥 + 𝑏)}
2 2 2
(As desired)

Problem 04: If 𝑦 = 𝑒 3𝑥 cos 4𝑥 then 𝒚𝒏 =?


Solution: 𝑦 = 𝑒 3𝑥 cos 4𝑥
⇒ 𝑦1 = 3𝑒 3𝑥 cos 4𝑥 − 4𝑒 3𝑥 sin 4𝑥 = 𝑒 3𝑥 [3 cos 4𝑥 − 4 sin 4𝑥]
Let 3 = 𝑟 cos 𝜃 and 4 = 𝑟 sin 𝜃. Then 32 + 42 = 𝑟 2 (cos2 𝜃 + sin2 𝜃) = 𝑟 2 ⇒ 𝑟 = 5
4
and 𝜃 = tan−1 (3)
Therefore, ⇒ 𝑦1 = 𝑒 3𝑥 [𝑟 cos 𝜃 cos 4𝑥 − 𝑟 sin 𝜃 sin 4𝑥] = 𝑟𝑒 3𝑥 cos(4𝑥 + 𝜃)
⇒ 𝑦2 = 𝑟𝑒 3𝑥 [3 cos(4𝑥 + 𝜃) − 4 sin(4𝑥 + 𝜃)]
= 𝑟 2 𝑒 3𝑥 cos(4𝑥 + 2𝜃) ; [applying the previous method]
4
Proceeding in this way we get,𝑦𝑛 = 𝑟 𝑛 𝑒 3𝑥 cos(4𝑥 + 𝑛𝜃) = 5𝑛 𝑒 3𝑥 cos (4𝑥 + 𝑛 tan−1 (3)).
(As desired)
x2  x 1
Problem 05: If y  3 then 𝒚𝒏 =?
x  x2  6x
x2  x 1
Solution: We have y  3
x  x2  6x
x2  x 1 x2  x  1 x2  x  1
 3  
x  x 2  6 x x  x 2  x  6  x  x  2  x  3
1 1 1 1 1 1
 .  .  .
6 x 2  x  2  3  x  3
1 1 1 1 1 1
 y1   . 2  .  .
6 x 2  x  2  3  x  3 2
2

1.2 1 1.2 1 1.2 1


 y2  . 3 .  .
6 x 2  x  2 3
3  x  3 3
1.2.3 1 1.2.3 1 1.2.3 1
 y3   . 4 .  .
6 x 2  x  2 4
3  x  3 4
Proceeding in this ways we get,
1 1 1 1 1 1 
yn   1 n !  . n 1  .  .
n
 (As desired)
 6 x 2  x  2 n 1
3  x  3n 1 
Try Yourself: Find 𝑦𝑛 for the followings,
1
1. y  cos(ax  b) 2. y  e3 x sin 4 x 3. y 
x  5x  6
2

Mohammad Abdul Halim, Lecturer in Mathematics, GED, DIU


Leibnitz’s theorem:
Statement: If the functions 𝑢 = 𝑢(𝑥) and 𝑣 = 𝑣(𝑥) are differentiable 𝑛 times with respect to 𝑥
then
(𝑢𝑣)𝑛 = 𝑢𝑛 𝑣 + 𝑛𝐶1 𝑢𝑛−1 𝑣1 + 𝑛𝐶2 𝑢𝑛−2 𝑣2 + 𝑛𝐶3 𝑢𝑛−3 𝑣3 + ⋯ + +𝑛𝐶𝑟 𝑢𝑛−𝑟 𝑣𝑟 + ⋯ + 𝑢𝑣𝑛 , where the
suffixes of 𝑢 and 𝑣 bears the order of the derivative.
1
Problem01: For y  easin x
prove that (1  x 2 ) yn  2  (2n  1) xyn 1  (n 2  a 2 ) yn  0 .
Solution:
Given function is,
1
y  e asin x

Differentiating both sides of above equation w.r (with respect) to x we get


1 a
y1  e a sin x

1  x2
a
y1  y 
1  x2
y1 1  x 2  ay

y12 1  x 2  a 2 y 2  [Squaring both sides]
Again, differentiating both sides of above equation w.r (with respect) to x we get
1  x  .2 y y
2
1 2  y12   0  2 x   a 2 .2 yy1
1  x  y
2
2  xy1  a 2 y
Applying the Leibnitz theorem in the above equation we have,
 1  x  y 
2
2
n
  xy1 n   a 2 y 
n

1  x  y
2
n2 
 C1  2 x  yn 1  nC2  2  yn  xyn 1  nC1 1 yn  a 2 yn
n

n(n  1)
1  x  y
2
n2  n  2 x  yn 1 
2
 2  yn   xyn 1  n 1 yn   a 2 yn
1  x  y
2
n2  2 xnyn 1  n(n  1) yn  xyn 1  nyn  a 2 yn
1  x  y
2
n2  2 xnyn 1  (n 2  n) yn  xyn 1  nyn  a 2 yn

1  x  y
2
n 2   2n  1 xyn 1  (n 2  n) yn  nyn  a 2 yn
1  x  y
2
n2   2n  1 xyn 1  (n 2  n) yn  nyn  a 2 yn  0

1  x  y
2
n2   2n  1 xyn 1  (n 2  n  n  a 2 ) yn  0

1  x  y
2
n2   2n  1 xyn 1  (n 2  a 2 ) yn  0 (As desired)

Mohammad Abdul Halim, Lecturer in Mathematics, GED, DIU


Problem02: For y  sin(msin 1 x) prove that (1  x 2 ) yn  2  (2n  1) xyn 1  (m 2  n 2 ) yn  0 .
Solution:
Given function is,
y  sin(msin 1 x)
Differentiating both sides of above equation w.r (with respect) to x we get
1
y1  cos(m sin 1 x)  m
1  x2
y1 1  x 2  m cos(m sin 1 x)
 
y12 1  x 2  m 2 cos 2 (m sin 1 x) [Squaring both sides]
 
y12 1  x 2  m 2 1  sin 2 (msin 1x)  

y12 1  x 2  m 2 1  y 2   
Again, differentiating both sides of above equation w.r (with respect) to x we get
 
1  x 2 .2 y1 y2  y12   0  2 x   m 2  0  2 yy1 

1  x  . y 2
2  xy1  m 2 y
Applying the Leibnitz theorem in the above equation we have,
 
1  x2  y2   xy1 n   m2 y 
n n

1  x  y 2
n2
n n
 
 C1  2 x  yn 1  C2  2  yn  xyn 1  nC1 1 yn  m 2 yn
n(n  1)
1  x  y 2
n2  n  2 x  yn 1 
2
 2  yn   xyn1  n 1 yn   m 2 yn
1  x  y 2
n2  2 xnyn 1  n(n  1) yn  xyn 1  nyn   m 2 yn

1  x  y 2
n2  2 xnyn 1  (n 2  n) yn  xyn 1  nyn  m 2 yn
1  x  y 2
n2   2n  1 xyn 1  (n 2  n) yn  nyn  m 2 yn  0

1  x  y 2
n2   2n  1 xyn 1  (n 2  n  n  m 2 ) yn  0

1  x  y 2
n2   2n  1 xyn 1  (n 2  m 2 ) yn  0

1  x  y 2
n 2   2n  1 xyn 1  (m 2  n 2 ) yn  0 (As desired)
Problem03: Prove that (1  x)2 yn2  (2n  1) 1  x  yn1  (n 2  1) yn  0 for y  cos ln(1  x) .
Solution:
Given function is,
y  cos ln(1  x)
Differentiating both sides of above equation w.r (with respect) to x we get
1
y1   sin ln(1  x)   (0  1)
1 x

Mohammad Abdul Halim, Lecturer in Mathematics, GED, DIU


1
y1   sin ln(1  x) 
1 x
y1 1  x    sin ln(1  x)
Again, differentiating both sides of above equation w.r (with respect) to x we get
1
1  x  . y2  y1   0  1   cos ln(1  x)   (0  1)
1 x
1
1  x  . y2  y1   cos ln(1  x) 
1 x
1  x  . y2  y1 1  x    cos ln(1  x)
2

1  x  . y2  y1 1  x    y
2

1  x  . y2  y1 1  x   y  0
2

Applying the Leibnitz theorem in the above equation we have,


1  x  .y  2
2
n
  y1 1  x  n   y n  0

1  x  yn 2  nC1 2 1  x    0  1 yn1  nC2  2  yn  1  x  yn1  nC1 1 yn   yn  0


2

1  x 
2

yn2  nC1 2 1  x   yn1  nC2  2  yn  1  x  yn1  nC1 1 yn  yn  0 
n(n  1)
1  x  yn  2  2n 1  x   yn 1   2  yn  1  x  yn1  nyn   yn  0
2

1  x  yn2  2n 1  x   yn1  n(n 1) yn  1  x  yn1  nyn   yn  0


2

1  x  yn2  2n 1  x   yn1  (n2  n) yn  1  x  yn1  nyn   yn  0


2

1  x  yn 2  2n 1  x   yn1  (n 2  n) yn  1  x  yn1  nyn  yn  0


2

1  x  yn 2   2n  11  x   yn1  (n 2  n) yn  nyn  yn  0


2

1  x  yn 2   2n  11  x   yn1  (n 2  n  n  1) yn  0


2

1  x  yn  2   2n  11  x   yn 1  (n 2  1) yn  0
2
(As desired)
Try Yourself:
1. For y  (a sin 1 bx)2 obtain a relational equation connecting yn  2 , yn 1 & yn .

2. For y   x  1  x 2  prove that (1  x 2 ) yn  2  (2n  1) x yn 1   n 2  m 2  yn  0 .


m

 
1
3. For y  emcos x
prove that (1  x 2 ) yn  2  (2n  1) xyn 1  (n 2  m 2 ) yn  0 .
4. For ln y  tan 1 x prove that (1  x 2 ) yn  2  (2nx  2 x  1) yn 1  n(n  1) yn  0 .

Mohammad Abdul Halim, Lecturer in Mathematics, GED, DIU


Chapter 06 : Maxima and Minima
Our objective in this section is to find the maximum and minimum values of a function using
derivative analysis.
Condition for having a maxima and minima of a function at a point:
𝑑𝑦 𝑑2 𝑦
 If 𝑦 = 𝑓(𝑥) then at 𝑥 = 𝑎 , 𝑓(𝑥) have a maxima when = 0 𝑎𝑛𝑑 < 0 at 𝑥 = 𝑎 .
𝑑𝑥 𝑑𝑥 2
𝑑𝑦 𝑑2 𝑦
 If 𝑦 = 𝑓(𝑥) then at 𝑥 = 𝑎 , 𝑓(𝑥) have a maxima when 𝑑𝑥
= 0 𝑎𝑛𝑑 𝑑𝑥 2
> 0 at 𝑥 = 𝑎 .
𝑑2 𝑦
 If 𝑑𝑥 2
= 0 at 𝑥 = 𝑎 , then for maxima and minima we have to determine higher derivative
more than 2
i) If odd order derivative is non-zero then 𝑓(𝑥) have no maxima and minima
ii) If even order derivative is non-zero and less than zero then 𝑓(𝑥).

Problem 01: Find the maximum and minimum values of y  x 5  5 x 4  5 x 3  10 .


Solution:
Let f ( x)  y  x 5  5 x 4  5 x 3  10
f ( x)  5 x 4  20 x 3 15 x 2
f ( x)  20 x 3  60 x 2  30 x

For maximum and minimum, f ( x)  0


 5 x 4  20 x 3 15 x 2  0
 x 4  4 x 3  3x 2  0

 x 2 x2  4 x 3 0 
 x  x  3 x  1 0
2

x  0 , 1, 3
When x  1, f (1)  20  60  30   10  0
Therefore, the given function is maximum when x  1 and maximum value f 1  1 5  5 10   9 .
When x  3, f (3)  540  540  90  90  0
Therefore, the given function is minimum when x  3
and the minimum value is f 3  243  40 5  13 5 10   37
When x  0, f (0)  0
Therefore the test fails.
 f ( x)  60 x 2 120 x  30
When x  0, f (0)  30  0
Therefore, the given function is neither maximum nor minimum when x  0 (Ans)
3
x
Problem 02:Find the extremum values of  a x 2  3a 2 x .
3
Solution:
x3
Let f ( x)   a x 2  3a 2 x
3

Mohammad Abdul Halim, Lecturer in Mathematics, GED, DIU


3x 2
f ( x)   2a x  3a 2
3
 f ( x)  x 2  2a x  3a 2
f ( x)  2 x  2a
For maximum and minimum, f ( x)  0
 x 2  2a x  3a 2  0
 x 2  3a x  ax  3a 2  0
 x( x  3a )  a( x  3a )  0
 ( x  3a )( x  a )  0
x  a ,  3 a
W hen x   3 a, f (3a)   6a  2a   4a  0
Therefore, the given function is maximum when x   3 a
and maximum value is f  3 a   9 a 3
W hen x  a, f (a)  2a  2a  4a  0

Therefore, the given function is minimum when x  a and minimum value is f a  


5 3
a (Ans)
3
Try Yourself:
1. Find the maximum and minimum values for f ( x)  3x 5  25 x 2  60 x .
2. Find the maximum and minimum values for f ( x)  x 6  12 x 5  36 x 4  4 .
Problem 03: An open box is to be made from a 16-inch by 30-inch piece of cardboard by
cutting out squares of equal size from the four corners and bending up the sides. What size
should the squares be to obtain a box with the largest volume?

30-2x

16 in
16-2x
x
x 30 inch

Let x be the length of the square to be cut out and V be the volume of the resulting box. Because
we are removing a square of side x from each corner, the resulting box will have dimensions
16  2x by 30  2x by x .
Since the volume of a box is the product of its dimensions, we have
V  16  2x 30  2x x
 4x 8  x 15  x
 480 x  92 x 2  4 x 3
Because x represents a length, it cannot be negative and because the width of the cardboard is 16
inches, we cannot cutout squares whose sides are more than 8 inches long. Thus the variable x
must satisfy 0  x  8 .

Mohammad Abdul Halim, Lecturer in Mathematics, GED, DIU


 480  184 x  12 x 2  4 x  12 3x  10
dV
Now
dx
 0 yields 4 x  12 3x  10  0
dV
Setting
dx
 x  12 3x  10  0
10
 x 12 and x 
3
Since x  12 falls outside the interval 0 , 8 the maximum value of V occurs either at the critical
10
number x  or at the endpoints x  0 , x  8 .
3

10
x 0 8
3

V  16  2x 30  2x x


19600
0 0
27

10 19600
Therefore maximum volume occurs when x  inches and maximum volume is .
3 27
Pr0blem 04: A garden is to be laid out in a rectangular area and protected by a chicken wire
fence. What is the largest possible area of the garden if only 100 running feet of chicken
wire is available for the fence?
Solution:

x
Let x (ft) be the length, y (ft) be the width and A (ft2) be the area of the rectangular garden.
Then A  xy … … … … … (i)
Since the perimeter of the rectangular garden is 100 ft., the variables x and y are related by the
equation
2 x  2 y  100
or , y  50  x … … … … … … (ii)
Substituting (ii) in (i)
A  x 50  x   50 x  x 2 … … … … … … … … … (iii)
Since x represents the length it can’t be negative and since the two sides of length x cannot have
a combined length exceeding the total perimeter of 100 ft, the variable x must satisfy 0  x  50
.
dA
Differentiating (iii) with respect to x we have,  50  2 x
dx

Mohammad Abdul Halim, Lecturer in Mathematics, GED, DIU


dA
Setting  0 we obtain 50  2 x  0 Or, x  25
dx
Thus the maximum occurs at one of the values x  0 , x  25 , x  50 .
Substituting these values in (iii) we get
x 0 25 50
A  x 50  x 0 625 0
 The maximum area is 625 ft2 occurs at x  25
Substituting x  25 in (ii) y  50  25  25
So the rectangle of perimeter 100 ft with greater area is a square with sides of length 25 ft.
Try Yourself:
1. For what value of x, the area of a rectangle with length 2x and breadth (15-2x) is
maximum?
2. The sum of the sides of a rectangle is constant .If the area is to be maximum. Show that
the rectangle must be a square.
3. Show that of all rectangles of given area, the square has the smallest perimeter.
4. Find two non-negative numbers whose sum is 10 and the sum of whose square is minimum.
5. A farmer has 800ft. of fencing with which to plants to enclose a rectangular pen. What
dimensions will maximize the possible area of such a rectangle.

Mohammad Abdul Halim, Lecturer in Mathematics, GED, DIU


Chapter 07 : Partial Differentiation
Differentiation is an operator that calculates the rate of change of dependent variable with respect
d
to independent variable and it is denoted by .In case of right angle triangle, if we denote the area
dx
1
by A , the height by x and the base by y then A  x y , we say that A is a function of x , y i.e.
2

A f x, y  .
Now if we want to calculate the rate at which the area of the triangle will increase if the height is
increase keeping the base constant, in that case the differentiation is called partial differentiation
A
and it is denoted by .
x
Similarly, if we want to calculate the rate at which the area of the triangle will increase if the base
A
is increase keeping the height constant, in that case the partial differentiation is denoted by .
y
So, in case of a function with more than one independent variable, to calculate the rate of change of
dependent variable with respect to only one independent variable keeping other independent
variable constant, we use partial differentiation.
2u 2u
 
Problem 01:If u  ln x 2  y 2 then show that 
x2 y2
0


Given, u  ln x 2  y 2 
u

x x

 
ln x 2  y 2   x 2
1
y2
. 2x

2u   2x  x 2

 y 2 2  2x . 2x 2y 2  2x2
  2 2 
  … … … … … (1)
x 2
x x y  x 2
y2  2
x 2
y2  2

u

y y

 
ln x 2  y 2   x 2
1
y2
.2y

2u   2y  x 2

 y 2 2  2y.2y 2x2  2y 2
  2 2 
  … … … … … (2)
y 2
y x y  x 2
y2  2
x 2
y2  2

Adding (1) and (2), we get


2u 2u 2y 2  2x2 2x2  2y 2
    0 (As desired)
x2 y2 x2y2
2
 x2y2
2
  
Problem 2: If u  e x y z then show that
3u
 x y z

 1  3 xyz  x 2 y 2 z 2 e x y z . 
Given, u  e x y z
u
 exyz xy
z

Mohammad Abdul Halim, Lecturer in Mathematics, GED, DIU


2u
 y z

 e x y z x  xy e x y z x z  e x y z x  x 2 y z 
3u
 x y z
 
 e x y z  1 2x y z   x  x 2 y z e x y z y z


 e x y z 1 2x y z  x y z  x 2 y 2 z 2 
 
 1  3 xyz  x 2 y 2 z 2 e x y z (As desired)
x u  2u 2
Problem-03: If u  x  y ln x  2e y then find
2 2
and 2 .
x 2 y

Sol : Giventhat , u  x 2  y 2 ln x  2e x y (1)


Differentiating (1) partially with respect to x we get,

u  2 y2
  x  y 2 ln x  2e  x y   2 x   2e  x y (2)
x x x
Now differentiating (2) partially with respect to x we get,

 2u   y2 x  y2
  2 x   2e y   2  2  2e x y (Ans.)
x 2
x  x  x

Again Differentiating (1) partially with respect to y we get,

u  2
  x  y 2 ln x  2e  x y   0  2 y ln x  2e x  2 y ln x  2e x (3)
y y
Now differentiating (3) partially with respect to y we get,

 2u 
  2 y ln x  2e x   2ln x  0  2 ln x (Ans.)
y 2
y

 2u  2u
Problem-04: If u  e  x cos y  y sin y  then find . Also show that  u2   u2  0.
x 2 2
and
x 2
y 2
x y

Sol : Giventhat , u  e x  x cos y  y sin y 


 xe x cos y  ye x sin y (1)
Differentiating (1) partially with respect to x we get,

u   
  xe x cos y  ye x sin y   cos y  xe x   y sin y  e x 
x x x x
 cos y  xe  e   ye sin y  xe cos y  e cos y  ye x sin y
x x x x x
(2)
Now differentiating (2) partially with respect to x we get,
 2u 
  xe x cos y  e x cos y  ye x sin y 
x 2
x

Mohammad Abdul Halim, Lecturer in Mathematics, GED, DIU


  xe x  e x  cos y  e x cos y  ye x sin y
 xe x cos y  2e x cos y  ye x sin y (3) (Ans.)
Again Differentiating (1) partially with respect to y we get,
u 

y y

xe x cos y  ye x sin y 
 
 xe x  cos y   e x  y sin y 
y y
 xe x   sin y   e x  y cos y  sin y 
  xe x sin y  ye x cos y  e x sin y (4)
Now differentiating (4) partially with respect to y we get,
 2u 

y 2 y

 xe x sin y  ye x cos y  e x sin y 
  xe x cos y  e x   y sin y  cos y   e x cos y
  xe x cos y  e x y sin y  e x cos y  e x cos y
  xe x cos y  e x y sin y  2e x cos y (5) (Ans.)
Finally, adding (3) and (5) we get,
 2u  2u
   xe x cos y  2e x cos y  ye x sin y     xe x cos y  e x y sin y  2e x cos y 
x 2 y 2
 xe x cos y  2e x cos y  ye x sin y  xe x cos y  e x y sin y  2e x cos y
 0 (Showed).
Symmetric Function:
A function u  f  x, y  is called a symmetric function if it satisfies the condition
f  x, y   f  y, x  .
Example: u  x 2  y 2 is a symmetric function.
 2u  2u  2u  2u
Problem-01: If u  x  3x y  3xy  y then find
3 2 2 3
, , and .
x 2 y 2 xy yx
Sol : Giventhat , u  x3  3x 2 y  3xy 2  y 3 (1)
Differentiating (1) partially with respect to x we get,
u  3
  x  3 x 2 y  3 xy 2  y 3   3x 2  6 xy  3 y 2  0
x x
u
  3 x 2  6 xy  3 y 2 (2)
x
Now differentiating (2) partially with respect to x we get,
 2u 
  3x 2  6 xy  3 y 2   6 x  6 y  0  6 x  6 y (Ans.)
x 2
x
Again Differentiating (1) partially with respect to y we get,
u  3
  x  3x 2 y  3xy 2  y 3   0  3x 2  6 xy  3 y 2
y y

Mohammad Abdul Halim, Lecturer in Mathematics, GED, DIU


u
  3x 2  6 xy  3 y 2 (3)
y
Now differentiating (3) partially with respect to y we get,
 2u 
  3x 2  6 xy  3 y 2   0  6 x  6 y  6 x  6 y (Ans.)
y 2
y
Again Differentiating (3) partially with respect to x we get,
 2u 
  3x 2  6 xy  3 y 2   6 x  6 y  0  6 x  6 y (Ans.)
xy x
Again Differentiating (2) partially with respect to y we get,
 2u 
  3x 2  6 xy  3 y 2   0  6 x  6 y  6 x  6 y (Ans.)
yx y
u u u
Problem-02: If u  x 2  y 2  z 2 then show that x  y  z  2u.
x y z
Sol : Giventhat , u  x  y  z
2 2 2
(1)
Differentiating (1) partially with respect to x we get,
u  2
  x  y 2  z 2   2x  0  0  2x
x x
u
x  2x2 (2)
x
Since the given function (1) is a symmetric function, so similarly differentiating (1) with respect to
y and z we get,
u
y  2 y2 (3)
y
u
and z  2z2 (4)
z
Finally adding (2), (3) and (4) we get,
u u u
x y z  2 x 2  2 y 2  2 z 2  2  x 2  y 2  z 2   2u (Showed.)
x y z
1
u u u
 

Problem-03: If u  x 2  y 2  z 2 2 then show that x  y  z  u.
x y z
1
Sol : Giventhat , u   x 2  y 2  z 2 

2 (1)
Differentiating (1) partially with respect to x we get,
u   2  
3

  x  y 2  z 2  2     x 2  y 2  z 2  2 .  x 2  y 2  z 2 
1
1 

x x   2 x
3 3


   x2  y 2  z 2  2 .  2 x  0  0    x x2  y 2  z 2 2 
1  

2
u 3
  x2  x2  y 2  z 2  2

x (2)
x

Mohammad Abdul Halim, Lecturer in Mathematics, GED, DIU


Since the given function (1) is a symmetric function, so similarly differentiating (1) with respect to
y and z we get,
u 3
  y 2  x2  y 2  z 2 

y 2 (3)
y
u 3
  z 2  x2  y 2  z 2 

and z 2 (4)
z
Finally adding (2), (3) and (4) we get,
u u u 3 3 3
  x2  x2  y 2  z 2   y 2  x2  y 2  z 2   z 2  x2  y 2  z 2 
  
x y z 2 2 2
x y z
3 1
  x  y  z  x    x  y  z 
 
2 2 2 2
y z
2 2 2 2 2 2 2  u (Showed.)
1
 2u  2u  2u 2

Problem-04: If u  x 2  y 2  z 2  2 then show that    .
x 2 y 2 z 2 u
1
Sol : Giventhat , u   x  y  z 2 2 2
 2 (1)
Differentiating (1) partially with respect to x we get,
u   2  1 2 1
 2
  x  y  z   x  y2  z2 
1
  x  y 2  z 2  2 2 
2 2 .
x x   2 x
1 1

   
1 2  
 x  y2  z2 2 .  2 x  0  0   x x2  y 2  z 2 2 (2)
2
Again Differentiating (2) partially with respect to x we get,
 2u   2 2
1

x 2 x 
 x  x 2
 y 2
 z  

 1 3
 1

   
 
 x.  x 2  y 2  z 2 2 .  2 x  0  0    x 2  y 2  z 2 2

 2 
3 1
  x2  x2  y 2  z 2    x2  y 2  z 2 
 
2 2

x2 1  x2  x2  y 2  z 2
 3
 1
 3

 x2  y 2  z 2  2
 x2  y 2  z 2  2
 x2  y 2  z 2  2

y z 2 2
 3
(3)
x 2
y z 2 2
 2

Since the given function (1) is a symmetric function, so similarly differentiating (1) with respect to
y and z we get,
 2u x2  z 2
 (4)
y 2 3

x 2
y z 2 2
 2

u 2
x y2 2
and  (5)
z 2 3

x 2
y z 2 2
 2

Mohammad Abdul Halim, Lecturer in Mathematics, GED, DIU


Finally adding (3), (4) and (5) we get,
 2u  2u  2u y2  z2 x2  z 2 x2  y 2
    
x 2 y 2 z 2 3 3 3

x 2
 y2  z2  x
2 2
 y2  z2  x  y  z 
2 2 2 2 2

y 2  z 2  x2  z 2  x2  y 2 2 x  y  z 
2 2 2

  3

x  y  z 
3

 x2  y 2  z 2  2 2 2 2 2

2 2
 1
 (Showed.)
x 2
y z
2 2
 2
u

Try Yourself:
 2u  2u  2u  2u
1. If u  e sin x cos x then find
xy
, , and
x 2 y 2 xy yx
u u u
2. If u  ln  x3  y 3  z 3  3xyz  then show that
3
   .
x y z x  y  z
1
 2u  2u  2u
3. If u   x  y  z  then show that
2 2 2 2
   0.
x 2 y 2 z 2
  2u  2u  2u 
4. If u  ln x 2
 y 2  z 2 then show that  x 2  y 2  z 2   2  2  2   1.
 x y z 
Homogeneous function:
A function f  x, y  is said to be homogeneous of degree n in the variables x and y if it can be
 y n  x 
expressed in the form x n   or y    .
x  y
Alternatively, a function f  x, y  is said to be homogeneous of degree n in the variables x and y if
f  tx , ty   t n f  x , y  for all values of t, where t is independent of x and y.
1
Example: f  x, y   x  y is a homogeneous function of degree .
2
Euler’s theorem on Homogeneous functions: If f  x, y  be a homogeneous function of x and y
of degree n, then
f f
x y  nf  x, y  .
x y
1  x3  y 3  u u
Problem-01: If u  tan   then show that x  y  sin 2u .
 x y  x y
 x3  y 3 
Sol : Giventhat , u  tan 1  
 x y 
 x3  y 3 
 tan u   
 x y 

Mohammad Abdul Halim, Lecturer in Mathematics, GED, DIU


  y 3 
x3 1    
  x    y
 tan u    tan u  x 2    say 
 y x
x 1  
 x
Here, tan u is a homogeneous function of degree 2.
By Euler’s Theorem we get,
 
x  tan u   y  tan u   2 tan u
x y
u u
 x sec2 u  y sec2 u  2 tan u
x y
u u 2 tan u
 x y 
x y sec2 u
u u
 x y  2sin u cos u
x y
u u
 x y  sin 2u (Showed).
x y
 x3  y 3  u u
Problem-02: If u  ln  2 2 
then show that x  y  1.
x y  x y
 x3  y 3 
Sol : Giventhat , u  ln  2 2 
x y 
  y  
3

x 3 1    
x3  y 3   x    y
 eu  2  eu    eu  x    say 
x y 2
  y  
2
x
x 2 1    
  x  
Here, eu is a homogeneous function of degree 1.
By Euler’s Theorem we get,
 u 
x
x
 e   y  eu   1
y
u u
 xeu  yeu  1.eu
x y
u u
 x y  1 (Showed).
x y
 x  u u u
Problem-03: If u  sin 1   then show that x  y z  0.
 yz x y z
 x 
Sol : Giventhat , u  sin 1  
 yz

Mohammad Abdul Halim, Lecturer in Mathematics, GED, DIU


1 1
x  yz y z
 sin u   sin u     sin u    
yz  x   x x
y z
 sin u  x 0  ,   say 
 x x
Here, sin u is a homogeneous function of degree 0.
By Euler’s Theorem we get,
  
x  sin u   y  sin u   z  sin u   0.sin u
x y z
u u u
 x cos u  y cos u  z cos u 0
x y z
u u u
 x y z 0 (Showed).
x y z
 x y  u u 1
Problem-04: If u  cos 1   then show that x  y  cot u  0 .
 x y x y 2

 x y 
Sol : Giventhat , u  cos1  
 x  y 

 y  y
x 1   x 1  
x y  x   cos u   x
 cos u   cos u 
x y  y  y
x 1   x 1  
 x  x
 y
1
 cos u  x 2    say 
x
1
Here, cos u is a homogeneous function of degree .
2
By Euler’s Theorem we get,
  1
x  cos u   y  cos u   cos u
x y 2
u u 1
  x sin u  y sin u   cos u
x y 2
u u 1
 x y   cot u
x y 2
u u 1
 x  y  cot u  0 (Showed).
x y 2
Try yourself:
 x2  y 2  u u 1
1. If u  tan 1   then show that x  y  sin 2u .
 x y  x y 2

Mohammad Abdul Halim, Lecturer in Mathematics, GED, DIU


 x y u u
2. If u  sin 1   then show that x y 0
 x  y  x y

 x y  u u 1
3. If u  sin 1   then show that x y  tan u
 x  y  x y 2

 x y  u u 1
4. If u  tan 1   then show that x y  sin 2u
 x  y  x y 4

Mohammad Abdul Halim, Lecturer in Mathematics, GED, DIU


Chapter 01 : Indefinite Integration
Integration: Integration is the process by which we can calculate the area of a zig-zag region by breaking the
entire region into regular well known pieces and adjoining those small areas.

In another way we define it as: “It is the process to find out the integral function which is called anti-derivative
process”.

 f ( x) dx  F ( x)  C

Integrand Integral function

Cycle of differentiation and Integration:


Integration

f (x ) F (x)

Differentiation

x2
For example: Let f ( x)  x then  f ( x) dx   x dx 
2
 C  F ( x) .

d d x2
After that {F ( x)}  (  C )  x  f ( x)
dx dx 2
It shows that integration is the inverse process of differentiation and vice versa.
Classification of Integration:
Integration are of two types such as
1. Indefinite Integration:
Indefinite integral represents a general formula to find out area enclosed by curves
2. Definite integration:
Definite integral represents the area enclosed by curves exactly.

Special Formulae for Indefinite Integral


dx 1 x
1. xa 22
 tan 1   where a  0
a a
dx 1 xa
2.  2  ln
x a 2
2a x  a
dx 1 ax
3.  2  ln
a x 2
2a a  x

4. 
dx
x a
2 2 
 ln x  x 2  a 2 

Mohammad Abdul Halim, Lecturer in Mathematics, GED, DIU


5.  x a
dx
2 2 
 ln x  x 2  a 2 
dx x
6. 
a2  x2
 sin 1  
a
dx 1  x
7.   sec1  
x x2  a2 a a
x x2  a2 a2
 x  a dx   ln x  x 2  a 2
2 2
8.
2 2
x x  a a2
2 2

 x  a dx   ln x  x2  a2
2 2
9.
2 2
x a2  x2 a2 x
10.  a 2  x 2 dx 
2
 sin 1  
2 a
 e ( f ( x)  f ( x))dx  e
x x
11. f ( x)

Indefinite Integral:

Simple Formula Based Problem

1  x3
 Evaluate the integral  1  x dx
Solution:
Given integral,
1  x3 1  x  (1  x  x 2 )
 1  x dx   1 x
dx   (1  x  x 2 ) dx   dx   xdx   x 2 dx

x 2 x3
 x  C (As desired)
2 3
1  x3  sin 3 x  cos3 x 
H.W:  dx ,    dx
1 x  sin x  cos x 
 e5ln x  e4ln x 
 Evaluate the integral   e3ln x  e2ln x  dx
Solution:
Given integral,
 e5ln x  e4ln x   eln x  eln x   x5  x 4  x 4  x  1
5 4

  e3ln x  e2ln x    eln x3  eln x2


dx   dx    3 2  dx   3
 x  x  x ( x  1)
dx
 
x2
  xdx   C (As desired)
2
 e e 
6ln x 3ln x
H.W:   e5ln x  e2ln x  dx

Mohammad Abdul Halim, Lecturer in Mathematics, GED, DIU


 Evaluate the integral  1  sin x dx
Solution:
Given integral,
2
x x x x  x x
 1  sin x dx   sin 2
2
 cos 2  2sin cos dx    sin  cos  dx
2 2 2  2 2
 x x x x
   sin  cos  dx   sin dx   cos dx
 2 2 2 2
x x
 cos sin
 2 2  C  2 cos x  2sin x  C (As desired)
1 1 2 2
2 2
H.W:  1  sin x dx ,  1  cos 2xdx .
 1  sin x 
 Evaluate the integral   1  sin x  dx
Solution:
Given integral is,
1  sin x  1  sin x 
2 2
 1  sin x 
  1  sin x  dx   1  sin x 1  sin x  dx   1  sin 2 x dx
 
1  sin x 
2
1  2sin x  sin 2 x
 dx   dx
cos 2 x cos 2 x
   sec 2 x  2 tan x.sec x  tan 2 x  dx


  sec 2 x  2 tan x.sec x  sec 2 x  1 dx 
   2sec 2

x  2 tan x.sec x  1 dx
 2 sec 2 xdx  2  tan x.sec xdx   dx
 2tan x  2sec x  x  C (As desired)
 Evaluate the integral  cos 4 x dx
Solution:
Given integral is,

 cos x dx  4  4 cos x dx  4   2 cos x  dx


4 1 4 1 2
2

  1  cos 2 x  dx   1  2 cos 2 x  cos2 2x  dx


1 2 1
4 4
1 1 1
  dx   cos 2 x dx   cos2 2 x dx
4 2 4
1 1 1
  dx   cos 2 x dx   2 cos 2 2 x dx
4 2 8
1 1 1
  dx   cos 2 x dx   1  cos 4 x  dx
4 2 8

Mohammad Abdul Halim, Lecturer in Mathematics, GED, DIU


x 1 sin 2 x 1 1
     dx   cos 4 x dx
4 2 2 8 8
x 1 sin 2 x x 1 sin 4 x
      C
4 2 2 8 8 4
3 x sin 2 x sin 4 x
   C (As desired)
8 4 32
 cos x dx ,  s in x dx
8 4
H.W:

cos 4 x  sin 4 x
 Evaluate the integral  (1  cos 4 x ) dx
Solution:
Given integral is,
cos 4 x  sin 4 x (cos 2 x  sin 2 x)(cos2 x  sin 2 x)
 (1  cos 4 x ) dx   (1  cos 4 x )
dx

cos 2 x  sin 2 x cos 2 x cos 2 x


 dx   dx   dx
(1  cos 4 x ) (1  cos 4 x ) 2 cos 2 2 x
cos 2 x 1 cos 2 x 1 x
 dx   dx   dx  C
2 cos 2 x 2 cos 2 x 2 2
sin x  cos x
 Evaluate the integral  dx
(1  sin 2 x )
Solution:
Given integral is,
sin x  cos x sin x  cos x
 (1  sin 2 x ) dx   (sin 2 x  cos2 x  2sin x cos x ) dx
sin x  cos x sin x  cos x
 dx   dx   dx  x  C
(sin x  cos x ) 2 sin x  cos x
sin 6 x  cos 6 x
 Evaluate the integral  sin 2 x cos2 x dx
Solution:
Given integral is,
 sin x    cos x 
2 3 2 3
sin 6 x  cos 6 x
 sin 2 x cos2 x dx  dx
sin 2 x cos 2 x
 sin x  cos 2 x   3sin 2 x cos 2 x  sin 2 x  cos 2 x 
2 3

 dx
sin 2 x cos 2 x
1  3sin 2 x cos 2 x  1 
 dx    2  3  dx
 sin x cos x 
2 2 2
sin x cos x
 sin x  cos x 
2 2
 1 1 
   3  dx      3  dx
 
2 2 2 2
 sin x cos x  cos x sin x

Mohammad Abdul Halim, Lecturer in Mathematics, GED, DIU


   sec 2 x  cos ec 2 x  3 dx  tan x  cot x  3x  C

H.W: Evaluate the following integral:

cos 2 x cos3 x  sin 3 x a cos3 x  b sin 3 x


1.  sin 2 x cos 2 x dx 2.  cos x  sin x dx 3.  sin 2 x cos2 x dx

 x 2  sin 2 x  2
 Evaluate the integral    sec x dx
 1  x 2

Solution:
Given integral is,
 x 2  sin 2 x  2  x 2  1  cos 2 x  2 (1  x 2 )sec 2 x  cos 2 x.sec 2 x
  1  x2  sec x dx    1  x2  sec x dx   1  x2
dx

(1  x 2 ) sec 2 x  1  1  1
 dx    sec 2 x  dx   sec x dx  
2
2 
dx
1 x 2
 1 x  1  x2
 tan x  tan 1 x  C (As desired)

 1 x 
2

 e  1  x 2  dx
x
 Evaluate the integral

Solution:
Given Integral is,
x  1 x  x  (1  x)  x 1  x  2 x  x  1 x 2x 
2 2 2 2

  1  x 2 
e dx    (1  x2 )2    (1  x2 )2    (1  x2 )2  (1  x2 )2  dx
e dx  e dx  e

 1 2x 
  ex   2 2
dx [  e x ( f ( x)  f ( x))dx  e x f ( x ) ]
 (1  x ) (1  x ) 
2

ex
 C (As desired)
1  x2
H.W:
xe x e x (1  x 2 ) e x (1  x)
1. dx 2. dx 3. dx
1  x  1  x  2  x
2 2 2

 Evaluate the integral  cos x cos 2 x cos 5 x dx


Solution:
Given Integral is,
1
 cos x cos 2 x cos 5 x dx  2  cos 2 x  2 cos x cos 5 x  dx
1 1
  cos 2 x  2 cos 5 x cos x  dx   cos 2 x  cos 6 x  cos 4 x  dx
2 2

Mohammad Abdul Halim, Lecturer in Mathematics, GED, DIU


1
 cos 6 x cos 2 x  cos 4 x cos 2 x  dx
2

1
  2  cos 6 x cos 2 x  cos 4 x cos 2 x  dx
4
1
   2 cos 6 x cos 2 x  2 cos 4 x cos 2 x  dx
4
1 1
  2 cos 6 x cos 2 x dx   2 cos 4 x cos 2 x dx
4 4
1 1
   cos8 x  cox 4 x  dx    cos 6 x  cox 2 x  dx
4 4
1 1 1 1
  cos8 x dx   cos 4 x dx   cos 6 x dx   cos 2 x dx
4 4 4 4
1 sin 8 x 1 sin 4 x 1 sin 6 x 1 sin 2 x
        C
4 8 4 4 4 6 4 2
sin 8 x sin 4 x sin 6 x sin 2 x
    C (As desired)
32 16 24 8

Change Variable/ Substitution Method


1
e tan x
 Evaluate the integral  dx
1  x2
Solution:
Given integral is,
1
e tan x
 1  x 2 dx
1
Say tan 1 x  z such that dx  dz
1  x2
1
e tan x
So,  dx   e z dz
1  x2
 ez  C
1
 etan x  C (As desired)
 Evaluate the integral  sin 4 x cos3 x dx
Solution:
Given integral is,
 sin x cos x dx   sin x cos x cos x dx   sin x 1  sin x  cos x dx
4 3 4 2 4 2

Say sin x  z such that cos x dx  dz


So,
  z 4 1  z 2  dz    z 4  z 6  dz   z 4 dx   z 6 dx

Mohammad Abdul Halim, Lecturer in Mathematics, GED, DIU


z5 z7 sin 5 x sin 7 x
  C   C (As desired)
5 6 5 6
e x (1  x)
 Evaluate the integral  dx
cos 2 ( xe x )
Solution:
Given integral is,
e x (1  x)
 cos2 ( xex ) dx
Say xe x  z such that e x 1  x  dx  dz


dz
2
cos ( z )
  sec 2 z dz  tan z  C  tan xe x  C   (As desired)

sin 2 x dx
 Evaluate the integral  a sin 2
x  b cos 2 x
.

Solution:
Given Integral is,
sin 2 x dx
 a sin2
x  b cos 2 x
Let a sin 2 x  b cos 2 x  z that implies
2a sin x cos x  2b cos x( sinx) dx  dz
a sin 2x  b sin 2 x) dx  dz
(a  b) sin 2 x dx  dz
dz
sin 2 x dx 
(a  b)
Now,
1
sin 2 x dx (a  b) 1 1 1
 a sin 2 x  b cos2 x   z dz  (a  b)  z dz  (a  b) ln z  C
1
 ln a sin 2 x  b cos 2 x  C
(a  b)
H.W: Evaluate the Integrals:
sin 2 x dx sin 2 x dx cos 2 x dx
1.  a sin2
x  b cos 2 x
2.  5sin 2
x  2 cos 2 x
3.  sin2
x  cos 2 x
tan x sec 2 x
 Evaluate the integral  dx
 
2
a 2  b 2 tan 2 x
Solution:
Given integral is,
tan x sec 2 x
 dx
 a 2  b2 tan 2 x 
2

Mohammad Abdul Halim, Lecturer in Mathematics, GED, DIU


dz
Say a 2  b 2 tan 2 x  z such that b 2 tan x sec 2 x dx  dz  tan x sec 2 x dx 
b2
1 dz 1 1 z 21 1 1
 z 2 b2  2 
  z 2
dz  dz   2  dz
b2 b 2  1 b z
1 1
  2 ln z + C   2 ln a 2  b 2 tan 2 x + C (As desired)
b b
dx
 Evaluate the integral  sin x cos 7 x
Solution:
Given integral is,
dx
 sin x cos 7 x

1
dx
 cos 4 x [Dividing numerator and denominator by
sin x cos7 x
cos 4 x
cos 4 x ]


sec4 xdx

sec2 x.sec2 xdx

1  tan 2 x  .sec2 xdx
tan x tan x tan x
Say tan x  z such that sec x dx  2 zdz
2 2

So,


1  z  .2 z dz
4
 z5 
 2  1  z 4  dz  2  z    C
z  5
 5

 2  tan x 
tan 2 x
C (As desired)
 
 3 
 
tan x
H.W:  sin x cos x dx
dx
 Evaluate the integral  3
dx
1  x  2 2

Solution:
Given integral is,
dx
 3
dx
1  x 2 2

Say x  tan  such that dx  sec2  d

Mohammad Abdul Halim, Lecturer in Mathematics, GED, DIU


So,

dx sec2  d sec2  d sec2  d d


 3
dx   3
 3 
sec 

sec 
1  x2  2 1  tan 2   2 sec2   2
3

  cos  d  sin   C  sin  tan 1 x   C (As desired)

1 x
 Evaluate the integral  cos{2cot 1 }dx
1 x
Solution:
Given integral is,
1 x
 cos{2cot
1
}dx
1 x
Say x  cos such that dx   sin  d
So,
1 x 1  cos 
 cos{2cot
1
}dx   cos{2cot 1 }  sin  d 
1 x 1  cos 

2 cos 2
  cos{2 cot 1 2 }   sin  d 
2
2sin
2

  cos{2cot 1 cot 2 }  sin  d 
2
 
  cos{2 cot 1 cot }   sin  d    cos{2. }   sin  d 
2 2
  cos    sin  d     sin  cos  d
1 1 1  cos 2
2 
 2sin  cos  d    sin 2 d   
2 2 2
C

cos 2 cos 2  cos 1 x 


 C   C (As desired)
4 4
sin x cos x
 Evaluate the integral  dx
cos 4 x  sin 4 x
Solution:
Given integral is,
sin x cos x sin x cos x tan x sec2 x
 cos 4 x  sin 4 x dx   4  sin 4 x  dx   1  tan 4 x  dx
cos x 1  4 
 cos x 
Putting tan 2 x  z that implies 2 tan x sec xdx  dz
2

 tan 1 z  c  tan 1  tan 2 x   C


1 dz 1 1
Now I 
2 1 z 2
2 2

Mohammad Abdul Halim, Lecturer in Mathematics, GED, DIU


Special types of formula based problem
x4
 Evaluate the integral  x10  4 dx
Solution:
Given integral is,
x4 x4
 x10  4 dx   x5 2  4 dx
 
Say z  x such that dz  5 x 4 dx
5

So,
x4 1 dz 1 dz 1 1 1 z 1 1 z
 x10  4 dx  5  z 2  4  5  z 2  22  5  2 tan a  C  10 tan a  C
1
 tan 1
x5
C
 (As desired)
10 a
dx
 Evaluate the integral  x  x
e e
Solution:
Given integral is,
dx e x dx
 e x  e x  e2 x  e0 [Multiplying numerator and denominator by e ]

x

e x dx

ex   1
2

Say z  e such that dz  e x dx


x

So,
 tan 1 z  C  tan 1  e x   C
dx dz
ee x
x
 2
z 1
(As desired)

dx
 Evaluate the integral 

1  x 9  (tan 1 x) 2
2
 
Solution:
Given integral is,
dx
 1  x 9  (tan
2 1
x) 2 
1 1
Say z  tan x such that dz  dx
1  x2
So,
dx dz dz 1 3  tan 1 x
 1  x 
2
9  (tan 1 x)2
  9  z 2  32  z 2 6 ln 3  tan 1 x  C
  (As desired)

Mohammad Abdul Halim, Lecturer in Mathematics, GED, DIU


dx
 Evaluate the integral  25 x 2  4
Solution:
Given integral is,
2
dx dx 1 dx 1 2
     ln x  x 2     C
25 x 2  4  4  5 2
2 5 5
25  x 2   x2   
 25  5
1 4
 ln x  x 2  C (As desired)
5 25
cos x
 Evaluate the integral  sin 2
x  4sin x  3
dx
Solution: Given integral is,

cos x
 sin
x  4sin x  3
dx
2

Say z  sin x such that dz  cos x dx


So,
cos x dz dz dz
 sin 2
x  4sin x  3
dx   2
z  4z  3
 2
z  4z  4 1

 z  2   12
2

1 z  2 1 1 z 1 1 sin x  1
 ln  C  ln  C  ln C (As desired)
2 z  2 1 2 z 3 2 sin x  3
dx
 Evaluate the integral x 2
 x 1
Solution: Given integral is,
dx dx dx
x 2
 x 1 

1 1 1
2 2

 1 1
2

x 2  2.x.        1  x    1
2 2 2  2 4
1
dx dx x
1
 2
 2  .tan 1 2 C
 1 3  1  3
2
3 3
x    x    
 2 4  2  2  2 2
2x 1

2
.tan 1 2  C  2 .tan 1 2 x  1  C (As desired)
3 3 3 3
2
H.W:
dx dx
1.  2x  x 1
2 2.  2x  5x  3
2

Mohammad Abdul Halim, Lecturer in Mathematics, GED, DIU


dx
 Evaluate the integral  x 2  7 x  12
Solution: Given integral is,
dx dx
 x  7 x  12
2

x  4 x  3 x  12
2

dx dx
 
x( x  4)  3( x  4) ( x  4)( x  3)
Say x  4  z such that dx  2 zdz
2

So,
dx 2 zdz 2 zdz 2dz dz
     2
x 2  7 x  12 z 2 ( z 2  4  3) z ( z 2  1) ( z 2  1) ( z 2  1)
 2 ln z  z 2  1  C  2ln x  4  x  4 1  C

 2ln x 4  x3 C (As desired)

H.W: Evaluate the Integrals:


dx  2x  1 
1. 
1 x  x 2
sin 1 
Ans:
 5 
C
dx  xa
2.  Ans: sin 1  C
2ax  x 2  a 
3x  2
 Evaluate the integral  dx
5x  2x  3
2

Solution:
Given integral is,
3x  2
 5 x 2  2 x  3 dx
3 3
10 x  2   2 
  10 2 5 dx
5x  2 x  3
3 7
10 x  2  
  10 2 5 dx
5x  2 x  3
3 10 x  2 7 dx
  2   2
10 5 x  2 x  3 5 5 x  2 x  3
3 7 dx f ( x)
 ln 5 x 2  2 x  3   2 [  dx  ln f ( x) ]
10 5 5x  2 x  3 f ( x)
3 7 dx
 ln 5 x 2  2 x  3  
10 5  2 2 3
5 x  x  
 5 5

Mohammad Abdul Halim, Lecturer in Mathematics, GED, DIU


3 7 dx
 ln 5 x 2  2 x  3  
10 5  2 2 3
5 x  x  
 5 5

3 7 dx
 ln 5 x 2  2 x  3  
10 25  2 1 1 3 1 
2 2

 x  2.x.        
 5 5 5 5 
3 7 dx
 ln 5 x 2  2 x  3   2
10 25  1 3 1
x   
 5  5 25
3 7 dx
 ln 5 x 2  2 x  3   2
10 25  1  14
x  
 5  25
3 7 dx
 ln 5 x 2  2 x  3   2
10 25  1   14 
2

x   
 5  5 
1
x
3 7 1 5 C
 ln 5 x 2  2 x  3   tan 1
10 25 14 14
5 5
5x  1
3 7 5 1 5 C
 ln 5 x  2 x  3  
2
tan
10 25 14 14
5
3 7 5x  1
 ln 5 x 2  2 x  3  tan 1 C (As desired)
10 5 14 14
H.W:
4  7x x2  x  1
1.  4  x2
2.  x2  x  1
dx

dx
 Evaluate the integral  ( x  4) x  3
Solution:
Given integral is,
dx
 ( x  4) x  3
Say x  3  z such that dx  2 zdz
2

So,

Mohammad Abdul Halim, Lecturer in Mathematics, GED, DIU


dx 2 zdz 2 zdz 2dz dz
   2  2  2
( x  4) x  3 ( z  7) z ( z  7)
 
2
( z 2  3  4) z 2 z2  7

1 z 7 1 x3  7
 2 ln C  ln C (As desired)
2. 7 z  7 7 x3  7
H.W:
dx dx
1. 
( x  3) x  2
2. 
( x  16) x  1
2

dx
 Evaluate the integral 
( x  3) x 2  6 x  8
Solution:
Given integral is,
dx
 ( x  3) x 2  6 x  8
1 2
Say x  3  such that dx   z dz
z

dx  z 2 dz z.z 2 dz
 ( x  3) x 2  6 x  8

2
 
1 1  1 
2
1  1 
.   3  6   3  8   3  6   3  8
z z  z  z  z 
2 2
z.z dz z.z dz
   
1 6 6 1
2
  9   18  8 1
z z z z2
z.z 2 dz z.z 2 dz z.z.z 2 dz
     
1 z2 1
. 1 z2 1 z2
z2 z

 cos 1 z  C  cos 1 
dz 1 
  C (As desired)
1 z 2
 x3
H.W:
dx dx
1.  ( x  1) 2 x 2  8 x  1
2.  ( x  1) 1  2 x  x 2
dx
 Evaluate the integral  ( x 2  1) x 2  4
Solution:
Given integral is,
dx
 ( x  1) x 2  4
2

Mohammad Abdul Halim, Lecturer in Mathematics, GED, DIU


1 2
Say x  such that dx   z dz
z
dx  z 2 dz  z 2 dz
 ( x2  1) x2  4   1  1
    1 z2  1 4z2

 2  2 1  4  2 
z  z  z  z2
 z 2 .z 2 dz zdz
  
1  z 2  . 1z 1  4 z 2 1  z 2  1  4 z 2
tdt
Again, Say 1  4z  t such that 4.2 zdz  2tdt  4 zdz  tdt  zdz 
2 2

4
dx 1 tdt 1 tdt
 ( x2  1) x2  4   4   t 2 1  2   4   4  t 2  1 
1   t  t
 4   4 
1 4tdt dt dt
    2  
4 4  t 1 t
2

t 3  t2  3  
2

1 1  4 z
2
1 1 t 1
 tan C   tan C
3 3 3 3
2
1 4
1 4  1 2
1  x 1
tan 1 x C
 tan 1 C  
3 3 3 3
x2  4 x2  4

1
tan 1 x 2  C   1 tan 1 x C
3 3 3 3
1 1 x2  4
 tan C (As desired)
3 x 3

H.W:
dx  x  5 dx
1.  (1  x )2
1 x 2
2.  (x 2
 2) x 2  3

dx
 Evaluate the integral  (1  x ) 2
1  x2
Solution:
dx cos  d
 (1  x ) 2
1  x2

(1  sin 2  ) 1  sin 2 
Let x  sin   dx  cos d

Mohammad Abdul Halim, Lecturer in Mathematics, GED, DIU


cos  d cos  d
 
(1  sin 2  ) cos 2  (1  sin 2  ) cos 
d

1  sin 2 
1
d
 cos 2
 [Dividing numerator and
1  sin 
2
denominator by cos  ]
2

cos 2 
sec2  d sec2  d

sec2   tan 2   1  tan 2   tan 2 

sec 2  d
 Let tan   z  sec  d  dz
2

1  2 tan 2 
dz dz
 2 
1 2z 1
2(  z 2 )
2
1 dz 1 dz 1 1 z
     . tan 1 C
2 ( 1  z 2 ) 2 {( 1 ) 2  z 2 } 2 1 1
2 2 2 2
1
 . tan 1 z 2  C
2

1
2

. tan 1 tan  2  C 

1
2

. tan 1  tan sin 1 x   2 C
1  x  
 . tan 1   tan tan 1   2   C
 1  x2
2   
1  x 
 . tan 1   2C
 1 x 
2
2
1  2x 
 . tan 1  C
2 
2  1 x 
(As desired)

dx
H.W:  (1  x )
2
1  x2

𝑑𝑥
 Evaluate ∫
4+5𝑠𝑖𝑛𝑥
dx
Sol n : Let I  
4  5sin x

Mohammad Abdul Halim, Lecturer in Mathematics, GED, DIU


dx dx
 
x x x
2 tan 4  4 tan 2  10 tan
4  5 2 2 2
x x
1  tan 2 1  tan 2
2 2
x
dx sec2 dx
  2
x
4  4 tan 2  10 tan
x  2 x x
2 2  4  4 tan  10 tan 
 2 2
2 x
sec
2
x x
Put tan  z  sec 2 dx  2dz
2 2
2dz 1 dz
Now I    
4  4 z  10 z
2
2  2 5 
 z  z  1
 2 
1 dz 1 dz
  2
  2 2
2 2 5 5 9 2  5 3
z  2. z      z  
  
4  4  16  4  4
  5 3  x 1
  z   tan 
ln  
1 1 4 4 1
    c  ln 2 2 c
2  2. 3  5 3  3 tan x  2
 4  z  4   4  2
 
H.W:
dx dx
1.  3  2 cos x 2.  2sin x  cos x  3
2sin x  3cos x
 Evaluate the integral  7 sin x  2 cos x
dx
Solution:
Given integral is,
2sin x  3cos x
 7 sin x  2 cos x
dx (i )
Suppose,
d
2sin x  3cos x  L  7 sin x  2 cos x   M 7 sin x  2 cos x 
dx
2sin x  3cos x  L   7 cos x  2sin x   M 7sin x  2cos x  (ii)
2sin x  3cos x   2 L  7 M  sin x  7 L  2M  cos x
Now equating like term on both sides we get,

2L  7M  2 and 7L  2M  3
Solving this two equations we get,

Mohammad Abdul Halim, Lecturer in Mathematics, GED, DIU


25 8
L &M 
53 53
Putting the values of L & M in the equations (i) we get
25 8
2sin x  3cos x    7 cos x  2sin x    7 sin x  2 cos x 
53 53
Using this relation we evaluate (i)
25 8
  7 cos x  2sin x    7 sin x  2 cos x 
2sin x  3cos x
 7 sin x  2 cos x
dx   53 53
7 sin x  2 cos x
dx
25 7 cos x  2sin x 8 7 sin x  2 cos x
  dx   dx
53 7 sin x  2 cos x 53 7 sin x  2 cos x
25 8
 ln 7 sin x  2 cos x   dx
53 53
25 8x
 ln 7 sin x  2 cos x   C (As desired)
53 53
H.W:
2 cos x  3sin x
 3cos x  2sin x
dx

Rational function based problem


x2  5x  7
 Evaluate the integral  ( x  1)( x  2)( x  4)
dx

Solution:
Given integral is,
x2  5x  7
 ( x  1)( x  2)( x  4)
dx
Now,
 1 7 11 
 5  
x2  5x  7
 ( x  1)( x  2)( x  4)
dx     6  30  dx
 x 1 x  2 x  4 
 
1 1 7 1 11 1
  dx   dx   dx
5 x 1 6 x2 30 x  4
1 7 11
 ln x  1  ln x  2  ln x  4  C
5 6 30
H.W:
( x  1)dx (7 x  4) dx
1.  ( x  1)( x  3)( x  5)
2.  x3  4 x

Mohammad Abdul Halim, Lecturer in Mathematics, GED, DIU


dx
 Evaluate the integral  ( x  9)( x 2  16)
2

Solution:
Given integral is,
dx
 ( x  9)( x 2  16)
2

Now,
 1 1 
dx  7 
 ( x2  9)( x2  16)    x2  9  x2 7 16  dx
 
 
 1 1 
 7  1 dx 1 dx
  2  2 7   dx   2   2
 x  16 x  9  7 x  16 7 x  9
 
1 dx 1 dx 1 1 x 1 1 x
  2 2   2 2   tan 1   tan 1  C
7 x 4 7 x 3 7 4 4 7 3 3
1 x 1 x
 tan 1  tan 1  C (As desired)
28 4 21 3
(2 x 2  1)dx
 Evaluate the integral 
( x  1)2 ( x  2)
Solution:
Given integral is,
(2 x 2  1)dx
 ( x  1)2 ( x  2) (i)
From (i) we get
1 7

(2 x 2  1) A 3  9
 
( x  1) ( x  2) x  1  x  12 x  2
2

(2 x 2  1) A 1 7
   (ii )
( x  1) ( x  2) x  1 3  x  1 9  x  2 
2 2

11
Putting above equation x  0 we get A 
9
In equation (ii) we get,

(2 x 2  1) 11 1 7
  
( x  1) ( x  2) 9  x  1 3  x  1 9  x  2 
2 2

Now from (i) we find

Mohammad Abdul Halim, Lecturer in Mathematics, GED, DIU


(2 x 2  1)dx  11 1 7 
 ( x  1) 2 ( x  2)   9  x  1 3  x  12 9  x  2  
     dx
 
11 dx 1 dx 7 dx
 
9 x  1 3   x  12 9  x  2
 

11 1 7
 ln x  1   ln x  2  C (As desired)
9 3  x  1 9
dx
 Evaluate the integral  x 1
3

Solution:
Given integral is,
dx
 x 1
3
(i )
From (i) we get
1 1 A Bx  C
   2 (ii ) [Say]
x  1 ( x  1)( x  x  1) x  1 x  x  1
3 2

 
1  A x 2  x  1   Bx  C  x  1
1  Ax  Ax  A  Bx 2  Cx  Bx  C
2

1   A  B  x 2   B  C  A x  A  C
Equating like term on both sides we get,
A  B  0 , B  C  A  0 and A  C  1
Solving these equations we get,
1 1 2
A , B   and C 
3 3 3

Using these values in equation (ii) we get,

1 x 2
 
1
 3  3 3
x3  1 x  1 x 2  x  1
1 1 x  2
 

x  1 3  x  1 3 x 2  x  1
3

1 1 x2
 
x  1 3  x  1 3 x  x  1
3 2
 
From (i) we get
dx  1 x2 
 x 1
3
   

 3  x  1 3 x 2  x  1 
 dx


Mohammad Abdul Halim, Lecturer in Mathematics, GED, DIU


dx 1 dx 1 x2
     2
x 1 3 x 1 3 x  x 1
3

1 1
1 2
2 x  1  2 
dx 1
 x3  1  3 ln x  1  3  x 2  x  1 2 dx
1 3
1 2
2 x  1 
dx 1
 x3  1  3 ln x  1  3  x 2  x  1 2 dx
dx 1 1 2x 1 1 dx
 x3  1  3 ln x  1  6  x 2  x  1 dx  2  x 2  x  1
dx 1 1 1 dx
 x3  1  3 ln x  1  6 ln x  x  1  2  x 2  x  1
2

dx 1 1 1 dx
 x3  1  3 ln x  1  6 ln x  x  1  2  2
2
2
1 1 1
x  2.x.     1 
2 2 4
dx 1 1 1 dx
 x3  1  3 ln x  1  6 ln x  x  1  2   1 2 3
2

x  
 2 4
dx 1 1 1 dx
 x3  1  3 ln x  1  6 ln x  x  1  2 
2
2
1  3
2

x   
 2  2 
1
x
dx 1 1 1 1
 x3  1  3 ln x  1  6 ln x  x  1  2  3 tan 32  C
2 1

2 2
dx 1 1 1 2x 1
  ln x  1  ln x 2  x  1 
x 1 3
3
6 3
tan 1
3
C (As desired)

Integration by parts Method


Technique of choosing first function in this method:
1. Both of two functions choose one of them second that is easy to integrate and
another one is first one.
2. If both of two one is algebraic then choose algebraic one is first function.

e
ax
 Evaluate cos bxdx
Sol n : Let I   e ax cos bxdx

Mohammad Abdul Halim, Lecturer in Mathematics, GED, DIU


d 
 e ax  cos bxdx     e ax   cos bxdx dx
 dx 
eax sin bx  sin bx 
   aeax dx
b  b 
eax sin bx a ax
   e sin bxdx
b b
e sin bx a  ax  d ax
   sin bxdx  dx 
ax
  e  sin bxdx    e
b b  dx 
eax sin bx a  eax cos bx a ax 
     e cos bxdx 
b b b b 
eax sin bx a ax a2
  2 e cos bx  2 I
b b b
2 ax
a e sin bx a ax
 I 2 I  2 e cos bx
b b b

I a b
2 2



be sin bx  aeax cos bx
ax

b2 b2
e ax  a cos bx  b sin bx 
I 
a 2  b2
e ax  a cos bx  b sin bx 
 I C
a 2  b2
Where c is an integrating constant.
H.W:
e ax  a sinbx  b cos bx 
e C
ax
Evaluate sin bx dx Ans:
a 2  b2
e 2 x  2sin 5 x  5cos 5 x 
e C
2x
Evaluate sin 5 x dx Ans:
29

 Evaluate x
2
cos xdx
Given Integral is,

x
2
cos xdx  x 2  cos x dx  
dx
    cos xdx dx
d 2
x

 x 2 sin x  2 x sin x dx

 
 x 2 sin x  2  x  sin x dx   1. sin x dx dx 
 
 x 2 sin x  2  x( sin x)   cos x dx 
 
 x sin x  2   x sin x  sin x  C
2

 x 2 sin x  2 x sin x  2sin x  C (As desired)

Mohammad Abdul Halim, Lecturer in Mathematics, GED, DIU


 Evaluate  ln(1  x 2 ) dx
Given Integral is,


 2x
ln(1  x 2 )dx  ln(1  x 2 )  dx   
1  x
2     2 x2 
   x ln(1  x )   1  x2 dx
dx dx 2

 x2   x2  1  1
 x ln(1  x 2 )  2  2
dx  x ln(1  x 2
)  2   1  x2 dx
1  x 
1  x 2 1   1 
 x ln(1  x 2 )  2   dx  x ln(1  x )  2  1 
2
2 2
dx
1  x 2
1  x   1  x 
1
 x ln(1  x 2 )  2  dx  2  dx  x ln(1  x 2 )  2 x  2 tan 1 x  C (Ans)
1  x2
H.W:
∫ ln(𝑥 + √(𝑥 2 + 𝑎2 )𝑑𝑥

  sin x 
2
1
 Evaluate dx
Given Integral is,
 
 sin x     dx   2sin 1
  dx dx
2 2
1
dx  sin 1 x 1
x.
 1  x2

 

 sin x   
x
dx  x sin 1 x   2sin 1 x.
2 2
1
dx

 1  x2 

1
Say sin 1 x  z such that dx  dz
1  x2
So,

  sin x  dx  x  sin x   2 z.sin z dz


2 2
1 1

  sin x  dx  x  sin x   2 z.sin z dz


2 2
1 1

 x  sin x   2  z.   cos z    1.   cos z  dz  C


2
1

 x  sin x   2sin x cos  sin x   2sin  sin x   C


2
1 1 1 1

 x  sin x   2sin x cos  sin x   2 x  C (As desired)


2
1 1 1

 1  x2 
 Evaluate  cos 1  2 
 1 x 
dx

Given Integral is,


1  1  x 
2

 cos  2 
 1 x 
dx

Say x  tan  such that dx  sec 2  d

Mohammad Abdul Halim, Lecturer in Mathematics, GED, DIU


So,

 1  x2  1  1  tan   2
2

   sec  d   cos  cos 2  sec  d


1
cos1  2 
dx  cos 
2

 1 x   1  tan 
2

  2 sec 2  d  2   sec 2  d  2   sec2  d   1.tan  d 


 

 
 2  tan   ln sec    C  2 tan 1 x tan tan 1 x  2 ln sec tan 1 x  C

 
 2 x tan 1 x  2ln sec tan 1 x  C (As desired)

x 2 sin 1 x
H.W:  dx
1  x 
3
2

 Evaluate x
2
sin x cos xdx
Given Integral is,
1 2
x sin x cos xdx 
2
2
x .2sin x cos xdx

1 d  
x sin 2 xdx   x 2  sin 2 xdx     x 2   sin 2 xdx  dx 
1 2

2  2  dx  

1  x2  x2 1  dx  
   cos 2 x   x cos 2 xdx    cos 2 x   x  cos 2 xdx     cos 2 xdx dx 
2 2  4 2  dx  

x2 1 x 1  x2 1 x 1 
 cos 2 x   sin 2 x   sin 2 xdx    cos 2 x   sin 2 x  cos 2 x   c
4 2 2 2  4 2 2 4 
x2 x 1
 cos 2 x  sin 2 x  cos 2 x  C (As desired)
4 4 8

Mohammad Abdul Halim, Lecturer in Mathematics, GED, DIU


Chapter 02 : Definite Integration
We already know that if F x  be any integral of f x  then

 f x  dx  F x   F b   F a 
b b
a a

 
The interval a, b is called the range of integration, a is called the lower limit and b is called the upper
limit of the definite integral.
Geometrical meaning:

 f x  dx y  f x  , the x -axis and the ordinate at x  a


b
represents the area enclosed between the curve
a

and x b i.e. the area PNMQ in the figure below.


Y

Q P

X
O M N

Fundamental theorem of Integral Calculus:

If f x  is continuous in a  x  b and
d
 F x  f x  then
dx
 f x  dx  F b   F a 
b

Even Function: A function f(x) is called even function if f ( x)   f ( x) .


For Example: y  f ( x)  x 2 is an even function.
Odd Function: A function f(x) is called even function if f ( x)  f ( x) .
For Example: y  f ( x)  x 3 is an odd function.

General Properties of the Definite Integral:

f  x  dx   f t  dt
b b
1.  a a

f x  dx    f x  dx
b a
2.  a b

f x  dx   f x  dx   f x  dx where c is any point in the interval  a, b .


b c b
3.  a a c

 f x  dx   f a  x  dx
a a
4.
0 0

Mohammad Abdul Halim, Lecturer in Mathematics, GED, DIU


0, when f x  is odd.

  a f x  dx  2 a f x  dx, when f x  is even.
a
5.
  0
0, if f 2a  x    f  x .

0 f x  dx  2 a f x  dx, if f 2a  x   f x .
2a
6.
 0

f  x  dx   f x  dx   f 2a  x .
2a a 2a
7.  0 0 0

f x  dx  n  f x  dx , if f x   f a  x 
na a
8.  0 0

f x  dx  n  f x  dx , if f x  ma   f  x 
na a
9.  0 0

0, when f x  is odd.



  a f x  dx  2 a f x  dx, when f x  is even.
a
 Prove That
  0
Proof:

  a f x  dx    a f x  dx   0 f x  dx … … … … … (i)
a 0 a

  a f x  dx    a f  t  dt
0 0
Now Putting x   t then d x   d t

f  t  d t
a
 When x   a, t  a and when x  0, t  0
0

f  x  d x
a

0

  a f x  dx   0 f  x  d x
0 a
Substituting in (i) we get

  a f x  dx   0 f  x  dx   0 f x  dx
a a a

When f x  is odd i.e. f  x   f x then

  a f x  dx    0 f x  dx   0 f x  dx 
a a a
0

When f x  is even i.e. f  x  f x then

  a f x  dx   0 f x  dx   0 f x  dx  2  0 f x  dx .
a a a a

Mohammad Abdul Halim, Lecturer in Mathematics, GED, DIU


3
dx
 Evaluate  ( x  2)
0 1 x
Given Integral is,
3
dx
 ( x  2)
0 1 x
Say 1  x  z such that dx  2 zdz .
2

Limit:
When x  0 then z  1
Again, When x  3 then z  2
Now,
3 2 2 2
dx 2 z dz dz dz
0 ( x  2) 1  x  1 z 2  1 z  21 z 2  1  21 z 2  1
   
 2  tan 2  tan 1
2
 2  tan 1 z  1 1
1

2 1 1
 2 tan 1  2 tan 1 (As desired)
1  2.1 3
1
dx
 Evaluate  (1  x)
0 1  2x  x2
Given Integral is,
1
dx
 (1  x)
1  2x  x2
0

1 1
Say 1  x  such that dx   2 dt .
t t
Limit:
When x  0 then t  1
1
Again, When x  1 then t 
2
1
1
1
2  dt
dx t2
Now,  (1  x) 1  2x  x2
  2
0 1 1 1  1 
1  2   1    1
t t  t 
1 1
2 2
dt dt
   
2 1 2  2 1 2
1  2  1
1
t 1   2   2   1 1
t
t t t  t t t

Mohammad Abdul Halim, Lecturer in Mathematics, GED, DIU


1 1 1 1
2 2 2 2
dt dt dt dt
       
1
t
4 1
 2 2 1 4t  1  2t 2 1 1  2t 2  4t 1 2  1  t 2  2t
t 2
t t t2
1 1 1
2 2 2
1 dt 1 dt 1 dt
     
2 1  1   t 2  2t  2 1 
1  1  t 2  2t  1 2 1 1   t  12
2
2 2
1
1    2
1 2
dt 1  1  t  1   1  1 1
   sin   sin 2  t  1  2
2  1   t 1 2
2
2  1 
  2 1
1
      2  1
 2


1  1
2 
sin
2 
2 1  1  sin 1 2 1  1   

1
2
sin 1 2  1
2  

1
2
sin 1 2 1 
2
1
2
sin 1
1
2
  (As desired)


2
dx
 Evaluate a
0
2
cos x  b2 sin 2 x
2

Given Integral is,


 
2 2
dx 1 dx 1 2
sec2 xdx
0 a2 cos2 x  b2 sin 2 x  b2   a  2
 b
 2  a 2

cos 2 x    tan 2 x     tan x


0 0 2

 b   b
Say tan x  t such that sec xdx  dt .
2

Limit:
When x  0 then t  0

Again, When x  then t  
2
 
1  1 
 
2
dx 1 dt 1 t 1
  2  b 1 bt 
Now,  2
a cos 2 x  b 2 sin 2 x b 2 0  a 2 2 b 2  a
  tan tan
a  b  a a  0
0
  t  b b 0
b

1  1  

1
ab
 tan 1   tan 1 0    tan tan  
ab  2  2ab
(As desired)

Mohammad Abdul Halim, Lecturer in Mathematics, GED, DIU



dx
 Evaluate  2  cos x
0
Given Integral is,
 x 2 x
   1  tan 2  dx
  sec dx
dx dx
  2

0 2  cos x  0
2
2 x 2 x 2 x x
1  tan 0 2  2 tan  1  tan 0 3  tan 2

2 2 2 2 2
2 x
1  tan
2
x 1 x x
Say tan  z such that sec2 dx  dz  sec 2 dx  2dz .
2 2 2 2

Limit:
When x  0 then z  0

Again, When x  then z  
2
     
dx 2dz dz dz  1 z  2  1 z 
0 2  cos x  0 3  z 2  20 3  z 2  2  2  tan 1    tan 
 
2
0 3 z 2  3 3  0 3  3 0

2   

2
3
 tan 1   tan 1 0     0 
32  3
(As desired)


dx
 Evaluates I    x      x  , (    )
  
Solution:
Given Integral is,


dx
I 
  x      x 
Say x    z such that dx  2 zdz .
2

Limit:
When x   then z  0
Again, When x   then z    
    
dx 2 zdz 2zdz
I     
  x      x  0 z    z
2 2
  0 z   z2 
   
dz dz
2 
0   z2  
2 
0    z2

Mohammad Abdul Halim, Lecturer in Mathematics, GED, DIU


   
   
 2  sin 1 1  sin 1 0   2   0 
dz z
2   2 sin 1 
 
    0 2 
2
0    z2
 (As desired)
𝜋
√𝑡𝑎𝑛𝑥
 Evaluate ∫02 𝑑𝑥
1+√𝑡𝑎𝑛𝑥
Solution:
𝜋
√𝑡𝑎𝑛𝑥
Let 𝐼 = ∫02 𝑑𝑥 ⋯ ⋯ ⋯ ⋯ ⋯ ⋯ ⋯ ⋯ ⋯ ⋯ (𝑖)
1+√𝑡𝑎𝑛𝑥
From the properties of definite Integral, we have

 f x  dx   f a  x  dx
a a

0 0
𝜋
𝜋 √tan( − 𝑥)
2
Therefore, 𝐼 = ∫02 𝜋
𝑑𝑥
1+√𝑡𝑎𝑛( − 𝑥)
2


cot x
I 2 dx
0
1  cot
1

I  2 tan x dx
0
1
1
tan x
1

I  2 tan x dx
0 1
1
tan x
1

I  2 tan x dx
0
1  tan x
tan x

1
I 2 dx (ii)
0
1  tan x
Adding equation (i) and (ii), we get
 
tan x 1
I I 2 dx   2 dx
0
1  tan x 0
1  tan x

1  tan x
2I   2 dx
0
1  tan x

2I   2 dx
0

2I   x 02
 
2I    0 
2 

Mohammad Abdul Halim, Lecturer in Mathematics, GED, DIU



I
4

tan x 
 2 dx 
0
1  tan x 4
 
 3  2 2
cos x cos x dx tan x dx
H.W: Evaluate  2
dx ,  2 dx ,  , dx ,
0 sin x  cos x 0
sin x  cos x 0
1  cot x 0 tan x  cot x

x dx
 1  sin x dx .
0
𝜋
 Evaluate ∫0 log(𝑠𝑖𝑛𝑥) 𝑑𝑥
2

Solution:

Let I   2 log  sin x  dx (i )
0
From the properties of definite Integral, we have

 f x  dx   f a  x  dx
a a

0 0

Therefore, I   2 log  sin x  dx
0

  
I   2 log  sin   x   dx
0
 2 

I   2 log  cos x  dx (ii)
0

Adding equation (i ) and (ii ) , we get


 
I  I   2 log  sin x  dx   2 log  cos x  dx
0 0

2 I   2 {log  sin x   log  cos x }dx
0

2 I   2 log  sin x cos x  dx
0

1 
2 I   2 log  .2.sin x cos x  dx
0
2 

1 
2 I   2 log  .sin 2 x  dx
0
2 

1
2 I   2 {log    log  sin 2 x } dx
0
2
 
1
2 I   2 log   dx   2 log  sin 2 x  dx
0
2 0

 
1
2 I  log    2 dx   2 log  sin 2 x  dx
2 0 0

Mohammad Abdul Halim, Lecturer in Mathematics, GED, DIU


 
1
2 I  log     x 02   2 log(sin 2 x) dx
2 0


 1   2
2 I  log      0    log(sin 2 x) dx
2  2  0

 
2 I   log1  log 2     0    2 log(sin 2 x) dx
2  0

 
2 I   0  log 2     0    2 log(sin 2 x) dx
2  0

 log 2
2 I   2 log(sin 2 x) dx 
0 2
 dz  log 2
2 I   log(sin z )  [let
0 2 2

2 x  z  2dx  dz lim it : x  0  z  0 x   z  ]
2
 dz  log 2
2 I   log(sin z ) 
0 2 2
1   log 2
2 I  .2 2 log(sin z ) dz  [
2 0 2
f x  dx  n  f x  dx , if f x   f a  x  ]
na a
 0 0

 log 2
 f x  dx   f t  dt ]
b b
2 I   2 log(sin x) dx  [
0 2 a a

 log 2
2I  I 
2
 log 2
 I 
2

 log 2
  2 log  sin x  dx   (As desired)
0 2

H.W: Evaluate 0
2
log(cos x) dx
1
dx
 Evaluate the integral  (1  x )
0
2
1  x2
Solution:
Given definite integral is,

cos  d
1 2
dx
 (1  x )
2
1  x2

(1  sin 2  ) 1  sin 2 
Let x  sin   dx  cos d
0 0 when x=0 then =0


2
cos  d and when x=1 then  
 2
0 (1  sin 2  ) cos 2 

Mohammad Abdul Halim, Lecturer in Mathematics, GED, DIU


 
2
cos  d 2
d
  
0
(1  sin  ) cos  0 1  sin 2 
2

 1
2 d
 cos 2  [Dividing numerator and
0
1  sin 
2
denominator by cos  ]
2

cos 2 
 
2
sec  d 2 2
sec2  d

sec2   tan 2  0 1  tan 2   tan 2 

0

2
sec2  d
 Let tan   z  sec 2  d  dz
0
1  2 tan 2 
 when =0 then z =0
2
dz 
 and when   then z  
0
1  2z2 2
 

dz 1 dz 122
dz
    
1
0 2 (  z2 )
2 0 (1  z2 ) 2 0 {( 1 )2  z 2 }
2 2 2

  
 1 1 z   1 1 
 . tan 1
  .tan 2 z 
2 1 1   2 0
 2 2  0
1  1  1 
 tan 2 z    tan 1     tan 1  0    (As desired)
2  0 2 2 2
1
dx
H.W:  Evaluate the integral  (1  x )
0
2
1  x2

2
dx
 Evaluate the integral  4  5sin x
0
Solution:
Given definite integral is,

 
2 2
dx dx
0 4  5sin x  0 x
2 tan
4  5 2
x
1  tan 2
2

Mohammad Abdul Halim, Lecturer in Mathematics, GED, DIU


 
2 2
dx dx
 
0 4  4 tan 2
x x 0 x x
 10 tan 4  4 tan 2  10 tan
2 2 2 2
x x
1  tan 2 sec2
2 2
 x
2 sec 2 dx x x
 2 Put tan  z  sec 2 dx  2dz
0  4  4 tan 2
x x 2 2
  10 tan 
 2 2
1 when x=0 then z =0
2dz
 
4  4 z 2
 10 z and when x  then z  1
0 2

1
1 dz 12 dz
 
20 2 5
 
 20 2 5 5
2
9
 z  z  1  z  2. z    
 2  4  4  16
1
   5 3 
1 1 z  
 4 4
2
1 dz  
   ln
20 2
5 3
2
2  2. 3  5 3 
 z      4 z  
 4 4   4 4 0
1
 x 1  1 1 0 1
tan  tan  tan 
1 2 2  1 2 2  ln 2 2}
 ln   {ln
3  tan x  2  3 1
tan  2
0
tan  2
 2  0 2 2
1 1
tan 
1 2 2 1
 {ln  ln } (Ans)
3 1 4
tan  2
2
   
2 2 2 2
dx dx dx dx
H.W: Evaluate the integral 0 5  3cos x , 0 3  2sin x , 0 3  2 tan x 0 3sin x  2 tan x  3
,

Mohammad Abdul Halim, Lecturer in Mathematics, GED, DIU


Chapter 03: Gamma and Beta Function
Gamma Function:

e
x
The integral x n 1 dx …………… (1)
0
is called the second Eulerian integral. The function (1) defined for positive value of n (n  0) is known as
Gamma function and denoted by  (n ) .

 (n)   e  x x n 1 dx
0
1

x
m 1
Beta Function: The integral (1  x) n 1 dx is known as the first Eulerian Integral. The above
0
function defined for positive values of m and n (m, n  0) is known as Beta function and is denoted by
B(m, n) .
1
 B(m, n)   x m1 (1  x) n1 dx
0

Properties of Beta Gamma function:


1. 11
1
2.  
2
3. n 1  n n  n
4.   m , n     n , m
m n
5.   m , n 
mn
 p 1 q 1
2
6.  sin x cos x dx  2 2
p q

0 pq2
2
2
 n 1
y
7. B ( m, n)   mn
dy
0
(1  y )

Mohammad Abdul Halim, Lecturer in Mathematics, GED, DIU


 p 1 q 1
2
 Prove that  sin p x cos q x dx  2 2
0 pq2
2
2
Proof:
We know that

  m , n    x m 1 (1  x) n 1 dx
0


Let x  sin
2
  dx  2sin  cos  d . Limit: x  0    0 and x      .
2
Now ,

2
  m , n    (sin 2  )m1 (1  sin 2  )n1  2sin  cos  d
0

2
  m , n    sin 2 m2  (cos2  )n1  2sin  cos  d
0

2
  m , n    sin 2m2  cos2n2   2sin  cos  d
0

2
  m , n   2  sin 2 m1  cos 2 n1  d
0
p 1 q 1
Assume 2 m  1  p and 2 n  1  q  m  2
and n 
2 .
Now from above equation we get

   2 sin
2
p 1
2
, q 1
2
p
 cos q  d
0

m n
Using the relation between beta and gamma function   m , n   ,we have
mn
p 1 q 1 
2
2
pq2 0
2  2 sin p  cos q  d

2
 p 1 q 1
2
  sin p  cos q  d  2 2 (Proved)
0 pq2
2
 Establish the relation between Gamma and Beta function.
 ( m) ( n)
Or Prove that  (m, n)  .
 ( m  n)

Mohammad Abdul Halim, Lecturer in Mathematics, GED, DIU


Solution:
From the definition of Gamma function we can write


(m)   e  x x m 1dx
0

Assume x  ky such that dx  kdy .


Limit:
When x  0 , then y  0 and when x   , then y   .
From above relation we have considering the transformation

(m)   e  ky ( ky ) m 1 kdy
0

  e  ky y m 1k m 1kdy
0

  e  ky y m 1k m dy (i )
0
Again,

(n)   e  k k n 1dk (ii )
0
Multiplying (i) and (ii) we get
 
(m)(n)   e  ky y m 1k m dy  e  k k n 1dk
0 0

(m)(n)    e  ky y m 1k m e  k k n 1dydk
0 0

(m)(n)    e  k  y 1 y m 1k m  n 1dydk
0 0

   k  y 1 m n1  m1



(m)(n)     e k dk  y dy
0 0 

 

(m)(n)     e k  y 1 k m n1dk  y m1dy
0 0 

 m  n  m1  n   kx n1 
(m)(n)    m n 
y dy  n   e x dx 
0
( y  1)   k 0 
 m 1
y
 ( m) ( n )  m  n  dy
0
( y  1) m  n
 
y n1 
(m)(n)  m  n   (m, n) 

 ( m, n )  
0
(1  y) m n
dy 

 ( m)  ( n )
  (m, n)  (Proved)
 ( m  n)

Mohammad Abdul Halim, Lecturer in Mathematics, GED, DIU



2

 cos
7
 Evaluate x dx
0
Solution:
Given Integral is
 
2 2

 cos x dx   cos x sin x dx


7 7 0

0 0

2
  sin 0 x cos7 x dx
0
Using the properties of gamma function, we get
0 1 7 1 1 8 1 1 1
4 3 1 3!
 2 2  2 2  2  2  2
072 9 9 7 7 5 3 1 1
2 2 2 2 1 2 . . . .
2 2 2 2 2 2 2 2 2
3.2.1 16
 
7 5 3 1 35
2 . . .
2 2 2 2

2
8
 sin xdx 
5
H.W: .
0
15
1
 Prove that ( )   .
2
Proof:
From the relation of beta and gamma functions we have
 ( m)  ( n )
B(m, n)  ………..(1)
 ( m  n)
1
Putting m  n  we get
2
1 1
 ( ) ( )
1 1 2 2
B( , ) 
2 2 1 1
(  )
2 2
1 1
1 1
1
1
1
 ( ) ( )
Or, x
0
2
(1  x) 2
dx  2 2
(1)

2
1 1
Or, ( )( )  2 d
2 2 0
[on putting x  sin 2  ]

Mohammad Abdul Halim, Lecturer in Mathematics, GED, DIU


1 1
Or, ( )( )  
2 2
1
 ( )   .
2
5
1
 x (1  x) dx
2
 Evaluate 2
0
Solution:
Given integral is
5
1
 0
x 2 (1  x) 2 dx
Let x  sin
2
  dx  2sin  cos  d .
Limit:

x  0    0 and x  1   
2

5 2 5
1
Now 0
x 2 (1  x) dx   (sin 2  )2 (1  sin 2  ) 2  2sin  cos  d
2

0

2 5
  sin 4  (cos2 ) 2  2sin  cos  d
0
 
2 2
  sin  cos   2sin  cos  d  2  sin 5  cos6  d
4 5

0 0

5 1 6 1 7 7
3 3
 2 2 2  2 2  2 2
56 2 13 11 9 7 7
. .
2 2 2 2 2 2
3 2.11 2
 2  2  2
11 9 7 11 9 7 11 9 7
. . . . . .
2 2 2 2 2 2 2 2 2
2 2
  (Ans)
77  9 693
3
π 1 3 3

H.W: Evaluate ∫02 Sin5 x Cos 6 x dx ,  x 2 (1  x) dx 


2

0
128
1
x
 Evaluate 
0 1  x2
dx

Solution:
Given integral is ,
1
x
 0 1  x2
dx

Say x  sin  such that dx  cos  d .

Mohammad Abdul Halim, Lecturer in Mathematics, GED, DIU



x  0    0 and x  1   
2
  
1
x 2
sin  2
sin  2

 dx   cos  d   cos  d   sin  d


0 1  x2 0 1  sin 2  0
cos  0

   cos  02   0  1  1.

Chapter 04 : Multiple Integral


2 3
 Evaluate x
2
y dx dy
1 0
Solution:
Given double integral is,
3
2 3
 x3  2
1
2
1
2

1 0 1  3  1   0 1   0 dy
3 3 3
x y dx2
dy   y  dy   x 3
y  dy  y  x
0
3 3
2
2 2 2
 y2 
  y  33  03  dy   27  y dy  9  y dy  9  
1 1
31 3 1 1  2 1
  y 2    22  12    3 
9 2 9 9 27
(As desired)
2 1 2 2 2
Note: It turns out that the result of two iterated integrals are always equal when the order of
integration is altered.
2 2
 Evaluate    x  3 y  dx dy
2

0 1
Solution:
Given double integral is,
2
2 2
 x2  2 2
 
   1
x  3 y dx dy     3 y 2 x  dy    2  6 y 2   3 y 2  dy
2

0 1 0 
0 1
2 2
2
3 y3   3 y
2
3  
2
    3 y 2  dy   y  3.     y 3 
0  2 3 0  2 0
2
  3  8  0  0   5 (As desired)
ln 2 1
 Evaluate   ye
xy
dx dy
0 1
Solution:
Given double integral is,
1
 e xy 
ln 2 1 ln 2 ln 2 ln 2 1 1
1
  ye dx dy   y   dy   y  e xy  dy   e xy  dy
xy

0 1 0  y  1 0
y 1 0 1

Mohammad Abdul Halim, Lecturer in Mathematics, GED, DIU


ln 2
ln 2 ln 2 ln 2
 e y 
 e  dy   e e
y y ln 2
 y
e y
dy  dy  e  y
 
 1  0
0
0 0 0

 e   e    e  e0    e  ln 2  e0    2  1  eln 2  1


0
y
0
ln 2  y ln 2 ln 2
 1


1 
 1   21  1  1    1  
1
(As desired)
2  2
 2
 Evaluate   y sin( xy)dx dy
0 1
Solution:
Given double integral is,
 2    2 2 2
  cos(xy)  1
0 1 y sin( xy)dx dy  0 y  y  dy  0 y  y cos(xy) dy  0 cos(xy) dy
1 1 1
  
    cos 2 y  cos y  dy   cos y dy   cos 2 y dy
0 0 0

 sin 2 y 
 sin y 0  

 2  0
1
  sin   sin 0    sin 2  sin 0   0 (As desired)
2
2 x
 Evaluate x
2
ydy dx
1 1 x
Solution:
Given double integral is,
x x
 y2 
 
2 x 2 2 2 2 2
x x
1 1x x ydy dx  1 x  2  dx  1 2  y  dx  1 2 x  1  x  dx
2 2 2 2

1 x 1 x

   
2 2
  x 2 x  1  x  dx   x 2 x  1  2 x  x 2  dx
1 2 1
21 21
2 2
x  x  1  2 x  x 2 dx    x3  x 2  2 x3  x 4 dx
1 2 1
 
21 21
2
1  x 4 x3 x5 
2
   3 x 3  x 2  x 4 dx  3.   
1
21 2  4 3 5 1
13 8 32 3 1 1  1  8 32 3 1 1 
  .16        12      
24 3 5 4 3 5 2 3 5 4 3 5
1 163 163
   (As desired)
2 60 120
H.W:
2 1
25
 Evaluate   ( x  y)
2
dydx Ans:
1 0
6

Mohammad Abdul Halim, Lecturer in Mathematics, GED, DIU


4 1
 Evaluate   xy( x  y) dydx
0 0
Ans: 8


2
 Evaluate   sin( x  y) dxdy
0 0
Ans: 2

1 x
25
 Evaluate   (x  y 2 ) dydx
2
Ans:
0 x
6
Triple Integration
2 z x 3
x
 Evaluate  
0 0 0
x  y2
2
dydxdz

Solution:
Given double integral is,
x 3 x 3
1 y  y
2 z x 3 2 z 2 z
x
 
0 0 0
x y
2 2
dydxdz    x  tan 1
0 0 
x 
x 0
dxdz     tan 1
0 0
x  0
dxdz


  
2 z 2 z
   tan 1
3  tan 0 dxdz      0 dxdz
1

0 0 
0 0
3
2 z
  2 z
 2
 2
 2
    dxdz    x0 dz    z  0 dz  3 0
z
dxdz  z dz
0 0
3 3 0 0
3 0
3 0
2
 z   2
2
       2  0  (As desired)
3  2 0 3 3
3a 2 a a
 Evaluate     x  y  z  dxdydz
0 0 0
Solution:
Given double integral is,
a
3a 2 a a
 x2 3a 2 a

 0 0  x  y  z  dxdydz  0 0  2  yx  zx dydz
0 0
3a 2 a
a 2
 3a 2 a
 a2 
     ya  za  0  dydz      ya  za  dydz
0 0   0 0  
2 2
2a
 a 2 y ay 2
3a
 3a

   zay  dz   a3  2a3  2a 2 z dz  


0  0
2 2 0
3a
 3 2 z 
3a

 
2
  3a  2a z dz  3a z  2a .   3a3 z  a 2 z 2 
3 2 3a

 2 0 0
0

  9a 4  9a 4  0   18a 4 (As desired)

Mohammad Abdul Halim, Lecturer in Mathematics, GED, DIU


H.W:
4 3 2
 Evaluate    3xy z
3 2
dzdxdy Ans: 2040
1 1 0
2 1 y
107
 Evaluate    xy z Ans: 
2 3
dxdzdy
0 y z2
210
3 1 xy
1
 Evaluate   xyz dzdydx Ans:  26  3ln 3
1 1 0
18
x
3a 2 a a
 Evaluate     x  y  z  dxdydz
0 0 0
Ans: 18a 4

3 1 2
 Evaluate     x  y  z  dzdydx
3 0 0
Ans: 12

END

Mohammad Abdul Halim, Lecturer in Mathematics, GED, DIU

You might also like